You are on page 1of 44

Chapter

5 Combinatorics

Hints and Explanations


Know Your Progress-1 From equation (i) and (ii)
(A∩B)′ = A′∪B′
1. {0, 3, 8, 15, 24, 35} Hence proved
We know that 12 – 1 = 0 5. Let A is the set of students who takes tea and B is the
22 – 1 = 3 set of students who take coffee
32 – 1 = 8 So n(A) = 26, n(B) = 18, n((A∪B)′] = 8
42 – 1 = 15 So n (A∪B) = N – n(A∪B)′)
52 – 1 = 24 = 40 – 8 = 32
62 – 1 = 36 Now n (A∪B) = n(A) + n(B) – n(A∩B)
So this set is also written as 32 = 26 + 18 – n(A∩B)
{x : x = n2 – 1, n ∈ N, n ≤ 6} n(A∩B) = 12
2. x ≥ 1, x ∈ N and 4x + 9 < 50 So 12 students take both tea and coffee.
So 4x < 41 6. Let A is the set of persons who drink tea and B is the
41 set of persons who drink coffee.
x< ⇒ x < 10.25
4 So n(A) = 30, n(A – B) = 14
So possible values of x So the persons who drink tea and coffee
= {1, 2, 3, 4, 5, 6, 7, 8, 9, 10} n (A∩B) = n(A) – n(A – B)
= 30 – 14 = 16
3. Given that A = {1, 2, 3, 4, 5}
Now persons who drink coffee but not tea = n(B – A)
B = {1, 3, 5, 8} and
= N – n(A)
C = {2, 5, 7, 8}
= 50 – 30 = 20
L.H.S = A – (B ∪ C)
So B ∪ C = {1, 2, 3, 5, 7, 8} and A – (B ∪ C) = {4} 7. (i) A∪B = {2, 3, 5, 6, 8}
Now R.H.S = (A – B) ∩ (A – C) (ii) A∩B = {3}
(A – B) = {2, 4} and (A – C) = {1, 3, 4} (iii) A – B = {2, 5}
So (A – B) ∩ (A – C) = {4} (iv) (A – B)′ = {1, 2, 5, 6, 8, 9, 10} AQ:
Please
So L.H.S = R.H.S = {4} [Change in Q (ii) C → B and answer also] check
Hence proved if this
8. A B sentence
4. Given that U = {1, 2, 3, 4, 5, 6, 7, 8, 9} is needed
A = {2, 4, 6, 8} 14 + x x 3x here.
B = {2, 3, 5, 7}
Now
(A∩B) = {2} Now if n(A) = n(B)
So (A∩B)′ = {1, 3, 4, 5, 6, 7, 8, 9} (i) 14 + x + x = x + 3x
Again A′ = {1, 3, 5, 7, 9} and B′ = {1, 4, 6, 8, 9} 14 = 2x
So A′∪B′ = {1, 3, 4, 5, 6, 7, 8, 9} (ii) x=7

M05 PMRO -Math Olympiad XXXX 00_Solution.indd 1 12-Oct-20 11:17:39 AM


5.2  Chapter 5

Know Your Progress-2 8. PROBLEM


For four letter words we can select 4 letters from all
3 2 7 letters and then arrange them
1. A B C
1 7× 6× 5× 4 × 3× 2×1
= 7C4 × 4 ! =
These are two ways from A to C 3× 2×1
(i) From A to B then B to C = 840
(ii) Direct A to C Now vowels are ‘O, E’
So total ways are So now we can arrange 2 letters in middle from 5 con-
3×2+1=7 sonents and 2 letters at the end from 2 vowels.
= 2C2 × 2 ! × 5C2 × 2 !
2. Method of selection of 2 balls are
= 40
(i) Both are red
(ii) 1 red and 1 green
(iii) Both are green
So total 3 ways
Know Your Progress-3
3. In first 100 natural numbers, 50 are odd and 50 are 1. We know that 3n things are to be divided equally
even. among three people then the number of ways in which
So selection of 4 even numbers from 50 even numbers
it can be done is
(3n ) !
= 50C4
(n !)3
4. First we arrange the letters of RANK according to So here 16 constables are divided equally in 8 group,
­dictionary A, K, N, R. so possible ways
Now number of words whose start from A = 3! = 6 16 !
Number of words whose start from K = 3! = 6 =
Numbers of words whose start from N = 3! = 6
(2 !)8
Now next word is RAKN 2. 6 = 1 + 2 + 3 (in different numbers)
Then next is RANK So first student select 1 book from 6 books then second
So rank of the word “RANK” = 3(6) + 1 + 1 student select 2 books from remaining 5 books and
= 20 then third student select all three books.
6C1 × 5C2 × 3C3
5. LEARN
Here 5 different letters, we can arrange the 5 letters Now these books can be distributed to 3 students by
by 5! way 3! ways
5! = 120 So possible combinations AQ:
Pleas
Now vowels are A and E, so we group a both vowels, = 6C1 × 5C2 × 3C3 × 3 !
check
then letters are → AE, L, R, N = 6 × 10 × 1 × 6 if thi
So total words in which both vowels are together = 360 sente
= 4! × 2 = 48 is nee
3. There are ‘n’ ways to choose the child that will not get here.
[We can also interchange ‘A’ and ‘E’ so multiplied by 2]
any toy.
6. When women are always included then remaining Again there are nC2 ways to choose 2 toys for a child
3 men are selected from 5 men. that will have two toys. There are (n – 1) ways to
5×4 choose the child. Who will get the 2 toys.
= 5C3 = = 10 ways There are (n – 2)! Ways to distribute the rest (n – 2)
2×1
toys to the rest (n – 2) children.
7. Exactly one even means remaining two are odd.
So possible ways to selecting 1 even from 10 even and So total ways = n × nC2 × (n - 1) × (n - 2) !
2 odd from 10 odd are = 10C1 × 10C2 n (n - 1)
= n× × (n - 1) (n - 2) !
10 × 9 1× 2
= 10 ×
  = 450 = nC2 × n!
2

M05 PMRO -Math Olympiad XXXX 00_Solution.indd 2 12-Oct-20 11:17:42 AM


Combinatorics    5.3

Know Your Progress-4 Know Your Progress-6


1. Number of numbers with n digits formed using first 5 1. First of all we construct a group of all 3 men. So now
natural numbers = 5n there are 4 members (3 women and 1 group of men’s)
Now the number of numbers which does not include 4 members are seated in round table by 3! ways.
2 = 4n. We can also arrange 3 men in group so total ways
Again similarly the number of numbers which does = 3! × 3! = 36
not include 4 = 4n
2. In necklace, clockwise and anticlockwise arrangements
Now the number of numbers which does not include
are same. So total ways in which 10 different diamonds
2 and 4 = 3n.
So the required number of numbers 9!
can be arranged = = 181440
= 5n – [4n + 4n – 3n] 2
= 5n – 4n – 4n + 3n 3. If we want 2 men are never together they we can select at
most 3 men so possible ways to selection of persons are
[1] 5 w 1m  [2] 4 w 2 m [3] 3 w 3 m
Know Your Progress-5 [1] For 5 w 1 m: First we select 5 women from 5 wom-
en and 1 men from 5 men. Then arrange 5 women
5! in round table, so there are 5 space between two
1. Total words = = 60
2! women for seating of men. Then we select 1 seat
Now arrange the letters according to alphabetically: for 1 men and then arrange them so total ways are
ADIIN 5C5 × 5C1 × 4 ! × 5C1 × 1! = 600
So total number of words starting with
4! [2] For 4 w 2 m
A – – – – – – – – – – = = 12 ⇒ 5C4 × 5C2 × 3 ! × 4C2 × 2 ! = 3600
2!
4! [3] For 3 w 3 m
D– – – – – – – – – – = = 12 5C3 × 5C3 × 2 ! × 3C3 × 3 ! = 1200
2!
I A – – – – – – – – – = 3! = 6 So total ways = 600 + 3600 + 1300
I D – – – – – – – – – = 3! = 6  = 5400
I I– – – – – – – – – – = 3! = 6 4. First we select 5 persons from 8 and then arrange on
I N A– – – – – – – – = 2! = 2 first table and then arrange remaining 3 persons on sec-
I N D A– – – – – – – = 1! = 1 ond table.
I N D I A– – – – – – = 1! = 1 So total ways = 8C5 × 4 ! × 2 ! = 2688
AQ:
Please
So rank of INDIA = 12 + 12 + 6 + 6 + 6 + 2 + 1 + 1
check   = 46
if this [no answer in answer key] Know Your Progress-7
sentence
is needed 2. 4-digit and 5-digit numbers are greater than 1000.
1. There are (p + 1) choices of each book. They are not
here. Now 4-digit can be formed by three ways in this
select any book, select 1 book, select 2 books … select
­question.
4! p books so total ways = (p + 1)n.
[1] From 1, 1, 2, 2 ⇒ =6 But we want at least one book so total ways decreases
2!2!
by 1, when none of the book is selected.
4! To finally ways = (p + 1)n – 1.
[2] From 1, 1, 2, 3 ⇒ = 12
2! 2. There are two choices for any question. Candidate give
4! the answer or not. So total way to candidate ­answer
[3] From 1, 2, 2, 3 ⇒ = 12
2! the question = 210. But candidate attempts at least one
5! question.
Now 5 digits numbers are = = 30 So ways are decreases by 1
2!2!
So total ways = 210 – 1
So total numbers = 6 + 12 + 12 + 30 = 60

M05 PMRO -Math Olympiad XXXX 00_Solution.indd 3 12-Oct-20 11:17:42 AM


5.4  Chapter 5

Know Your Progress-8 (ii) Total ways = 2C1 × 3C1 = 6


Sample space = {m1w1, m2w1, m1w2, m2w2, m1w3,
1. Let x, y, z and w be the number of apples given to the m2w3}
children so (iii) If head comes up, a die is thrown, so total 6 possi-
x + y + z + w = 12 bilities. But if tail comes up than the coin is tossed
(a) Here 0 ≤ x ≤ 12, 0 ≤ y ≤ 12, 0 ≤ z ≤ 12, 0 ≤ w ≤ 12 again so 2 possibilities.
So total ways = 12 + 4 - 1C4-1 = 15C3 Sample space = {H1, H2, H3, H4, H5, H6, TT, TH}
(b) If each boy receives at least 2 apples then (iv) Sample space of A = {2, 4, 6}
x ≥ 2, y ≥ 2, z ≥ 2, w ≥ 2 Sample space of B = {2, 3, 5}
So x = x′ + 2, y = y′ + 2, z = z′ + 2, w = w′ + 2 3 1 3 1
So P(A) = = and P(B) = =
Put the value in equation 6 2 6 2
(x′ + 2) + (y′ + 2) + (z′ + 2) + (w′ + 2) = 12 So A and B are equally likely events (A)
x + y′ + z′ + w′ = 4
2. (i) Total outcomes = 23 = 8
(here x′, y′, z′, w′ ≥ 0)
Favourable outcomes = HTT, THT, TTH, HHT,
So total ways = 4 + 4 - 1C4-1 = 7C3
HTH, THH (total 6)
2. x + y + z = 10 6 3
Here x ≥ 0, y ≥ 0, z ≥ 0 So probability = =
8 4
So total ways = 10 + 3 - 1C3-1 = 12C2 (ii) Total ways to selecting 4 balls from total 9 balls =
3. x + y + z = 20 9C4 = 126
x ≥ –4, y ≥ +1, z ≥ 2 Now selection of four balls in which 2 are red and
Lets take another variables x′, y′, z′ such that 2 are green = 5C2 × 4C2 = 10 × 6 = 60
x′ = x + 4,   y′ = y – 1,   z′ = z – 2
So x = x′ – 4,   y = y′ + 1,   z = z′ + 2 60 10
So probability = =
Put the value in equation 126 21
(x′ – 4) + (y′ + 1) (z′ + 2) = 20 (iii) There are 10 possible remainder when sum of two
x′ + y′ + z′ = 21 Here x′, y′, z′ ≥ 0 natural numbers is divided by 10. Number is divis-
So total ways = 21 + 3 - 1C3-1 = 23C2 ible by 10, if remainder is 0. So there are 1 choice
from 10.
1
Know Your Progress-9 So probability =
10
(iv) There are 13 cards of each four shade in pack of
1. There are 4 boxes and 4 balls. If we want the balls, 52 cards
one in each box, could be placed in such a way that a  So selection of 5 cards from these cards with
ball does not go to box of its own colour, then we de-­ ­replacement = 135.
arrange the 4 things.  Now selection of cards with at least one Ace
⎡ 1 1 1 1⎤ = 135 – 125
So no. of ways = 4 ! ⎢1 - + - + ⎥ = 9
⎣ 1! 2! 3! 4!⎦ 135 - 125
So probability =
135
Know Your Progress-10 3. (i) (a) (AC∩BC) ∪ (A∩B)
  Venn diagram of (AC∩BC)
1. (i) Let 2 red balls are R1 and R2 and 3 black balls are
A B
B1, B2 and B3.
Selection of two balls from total 5 balls are in 5C2
= 10 ways
So sample space = B  1R1, B2R1, B3R1, B1R2, B2R2,
B3R2, B1R2, B2B3, B1B3, R1R2}

M05 PMRO -Math Olympiad XXXX 00_Solution.indd 4 12-Oct-20 11:17:44 AM


Combinatorics    5.5

  Venn diagram of A∩B (iv) Let E is the set of those students who passed in En-
glish and M is the set of those students who passed
A B in maths.
So n(E) = 70, n(M) = 55 and n(E∩M) = 30
So n(E∪M) = n(E) + n(M) – n(E∩M)
  = 70 + 55 –30
  So Venn diagram of (AC∩BC) ∪ (A∩B)   = 95
(a) So probability that a student select at random
A B from the class has passed in at least one subject.
n (E ∪ M ) 95 19
= = =
N 125 25
(b) 
Probability that a student select at random
(i) (b) BC ∪ (AC∩B) from the class key passed in only one subject
  Venn diagram, of (AC∩B)
n (E ) + n ( M ) - 2n (E ∩ M )
=
A B N
70 + 55 - 2 (30 ) 65 13
= = =
125 125 25

  So Venn diagram of BC ∪ (AC∩B)


Exercise – 1 Part – 1 NSEJS Level
A B
Problems
1. A ∩ (A∪B)′
By de Morgan’s Law
(ii) If A and B are two mutually exclusive events than A ∩ (A′∩B′)
P(A∩B) = 0 By associative Law
Now P(A) + P(B) ≤ 1 (A∩A′) ∩ B′
P(A) ≤ 1 – P(B) = ϕ ∩ B′
P(A) ≤ P (B ) (i) =ϕ
Again
2. We know that
P ( A ∩ B ) = 1 – P(A) – P(B) A∪A′ = ∪
and A∩A′ = ϕ
= P ( A) – P(B)(ii)
3. B′ = {1, 2, 3, 4, 5, 8, 9, 10}
Again
A = {1, 2, 5}
P ( A ∪ B ) = P ( A) + P (B ) - P ( A ∩ B ) So A∩B′ = {1, 2, 5} = A
= 1 (iii) 4. If A∪B = A∩B then
And for P ( A ∩ B ) , P ( A) is consists P(B) A=B
So P ( A ∩ B ) = P(B)(iv) 5. A B
So from (i), (ii), (iii), (iv) options (A), (B), (C) and
(D) are correct.
(iii) Total balls = 6 + 5 + 4 = 15
Total white and black balls = 11 A−B = A∩B′
11
So probability =
5

M05 PMRO -Math Olympiad XXXX 00_Solution.indd 5 12-Oct-20 11:17:47 AM


5.6  Chapter 5

6. Two sets are disjoint iff: none of the elements in A and So B = {0} not a null set
B are common. C have one element ϕ, so it is also not a null set and if
Means A∩B = ϕ x ≥ 1 and x ≤ 1 than 1 is the solution of both inequal-
ities. So D have 1 element. So it is also not a null set.
7. If A ⊆ B then
So option (A) is correct.
By Venn diagram:
17. P(A) = P(B)
P(A) is a power set of A, means P(A) is set of all subsets
A B
of A. So
A ⊂ P(A)
So A ⊂ P(B)[∵ P(A) = P(B)]
So A∩B = A So A ⊂ B  (i)
Again similarly
8. A ∪ (A∩B) B ⊂ P(B)
By distributive Law B ⊂ P(A)[∵ P(B) = P(A)]
(A∪A) ∩ (A∪B) So B ⊂ A  (ii)
A ∩ (A∪B) From equation (i) and (ii)
A A = B
9. If A and B are not disjoint then A∩B ≠ ϕ 18. Washing Refrigerator
So n(A∪B) = n(A) + n(B) – n(A∩B) machine
a b c
10. A = {2, 4, 5} ⇒ 3 elements d
B = {7, 8, 9} ⇒ 3 elements e f
(A × B) ⇒ {(2, 7), (2, 8), (2, 9), (4, 7), (4, 8), (4, 9), (5, g
7), (5, 8), (5, 9)}
Total = 3 × 3 = 9 elements Vacuum cleaner
So according to given conditions
11. We know that a + e + g = 400
n(A∪B) = n(A) + n(B) – n(A∩B) a + b + c = 380
110 = 70 + 60 – n(A∩B) c + f + g = 542
n(A∩B) = 20 e + d = 294
12. ϕ = {} = null set is the subset of all given sets d + f = 277
b + d = 120
⎧ 1 ⎫ and a + b + c + d + e + f + g = 1000
13. Q = ⎨x; x = , when y ∈ N ⎬ (a + e ˙+ g)˙ + (b +˙d) + ˙ (c + f  )˙ = 1000
⎩ y ⎭
So in other form 400 + 120 + (542 – g) = 1000
g = 62
⎧1 1 1 1 ⎫ So 62 person had only a vacuum cleaner
Q = ⎨ , , , …⎬
⎩1 2 3 4 ⎭ Maths Physics
So that 1 ∈ Q
19. a b c
14. Every number is equal to itself e
d f
So A is the null set.
g
15. 3 is an element of set {1, 3, 5} So 3 ∈ {1, 3, 5}
But if write {3} than {3} is a subset of {1, 3, 5} so {3} Chemistry
⊆ {1, 3, 5} According to given conditions
So only (B) is correct
a + b + e + d = 37 (i)
16. If x > 1 and x > 1 so no value of x satisfy both inequal-   b + c + f + e = 24 (ii)
ities so option A is null set. d + e + f + g = 43 (iii)
Now if x + 3 = 3 then x = 0 b + e ≤ 19 (iv)

M05 PMRO -Math Olympiad XXXX 00_Solution.indd 6 12-Oct-20 11:17:47 AM


Combinatorics    5.7

d + e ≤ 29 (v) At all, we will convert all the values in terms of x.


e + f ≤ 20 (vi) Since the number of people cannot be negative
and a + b + c + d + e + f + g = 50 (vii) So x – 10 ≥ 0
From (i) + (ii) + (iii) – (iv) – (v) – (vi) we get x ≥ 10 (i)
(a + c + g) + (b + d + f) ≥ 37 + 24 + 43 – 19 – 29 –20 and 30 – x ≥ 0
(50 – e) ≥ 36 30 ≥ x
14 ≥ e So maximum number of people playing all three games
So largest possible number that could have passed all is 30.
three examination is 14. Now for minimum number of people playing at least
one game x = 10 and no. of people playing at least one
20. A = {(a, b) : a2 + 3b2 = 28, a, b ∈ z}
game is
B = {(a, b) : a > b, a, b ∈ z}
= x + 40 – x + x – 10 + x + 30 – x + 30 – x + 20 + x
We know that
= 110 + x
52 + 3(1)2 = 28
Put x = 10
42 + 3(2)2 = 28
So 110 + x = 120
12 + 3(3)3 = 28
So option (c) is correct.
So A = {(5, 1), (4, 2), (1, 3), (5, –1), (4, –2), (1, –3),
(–5, 1), (–4, 2), (–1, 3), (–5, –1), (–4, –2), 23. We have four groups. Each pair of groups has ­exactly
(–1, –3)} one students in common and each student belongs to
Total elements in A = 12 exactly two groups. So first group students is also com-
In which in half pairs a > b and in half pairs b > a, So mon with second, third and fourth group with exactly
In set A∩B, there are 6 elements 1 student so first group have at least 3 students. Let 3
Mathematics Statistics
students are a, b, c.
21.
Group 1 Group 2 Group 3 Group 4
a a b c
a b c b
c
Now group 2 is also common with group 3 and 4 with
According to given conditions
exactly one student. So
a + b = 12
Group 1 Group 2 Group 3 Group 4
a = 8
a a b c
a + b + c = 25
b d d e
So c = 25 – 12 = 13
c e
and b=4
Group 3 also common with exactly one student with
So, difference in the number of students who have tak-
group 4 so
en mathematics and statistics and those who have taken
Group 1 Group 2 Group 3 Group 4
statistics but not maths
a a b c
=b–c
b d d e
= 4 – 13
c e f f
= –9
So total we have atleast students.
Difference is taken always positive so it is 9
So n = 6
22. Cricket Hockey
(70) (60) 24. A = {1, 2, 3, 4}
If 3 is contained the 3 numbers are remaining {1, 2, 4}
x 40-x x-10 So number of subsets made by {1, 2, 4} are 23 = 8
x So finally number of subsets of set A containing ele-
30-x 30-x
ment 3 is also 8.
20+x
25. Set A∪B can be minimum number of elements if A∩B
have maximum number of elements. So for maximum
Football
value of A∩B, we consider all elements of A are in B
(80)
then, n(A∩B) = 3 and then n(A∪B) = B = 6 elements

M05 PMRO -Math Olympiad XXXX 00_Solution.indd 7 12-Oct-20 11:17:48 AM


5.8  Chapter 5

26. Let the number of news papers are ‘x’ then From equation (i), (ii) and (iii) we get
Total number of times that all news papers are read by b = 30 – 18 = 12, e = 28 – 18 = 10, f = 23 – 18 = 5
all students = 300 × 5 = 60 × x From equation (iv) a = 100 – 18 – 12 – 10 = 60
So    x = 25 From equation (v) c = 70 – 18 – 12 – 5 = 35
From equation (vi) g = 40 – 10 – 18 – 5 = 7
27. All shaded part is consist by A but not by B and C. So if
So a = 60 students have offered mathematics alone.
we subtract the B∪C from A, then we get shaded part.
Shaded part = A – (B∪C) 32. A∪B = A∪C
A + B – (A∩B) = A + C – (A∩C)
28. n(A∪B) = n(A) + n(B) – n(A∩B)
B – (A∩B) = C – (A∩C)
   = 200 + 300 – 100
Now again given
n(A∪B) = 400
A∩B = A∩C so
Again n(A′∩B′) = n(A∪B)′
 B = C
= n(U) – n(A∪B)
= 700 – 400 33. 2 x - 3 + x ( )
x -6 + 6 = 0
= 300
Case: 1 when x > 3  ⇒ x > 9
29. A = {1, 2, 3, 4, 5}
Total subsets = 25 = 32
(
2 x -3 +x -6 x + 6 = 0 )
But a null set, is also counted in 32 subsets. x – 4 x = 0
So proper subsets = 32 – 1 = 31

x ( )
x -4 = 0
30. Given equations are
x2 + y2 = 25 x = 0 (reject), x = 4 (select)
x2 + 9y2 = 144 x = 16
So by solving these equations we get Case 2 when 0 < x < 3

81
x= ± and y = ±
119
-2 ( )
x -3 +x -6 x +6 = 0
8 8 x - 8 x + 12 = 0

So there are four ordered pairs of (x, y) satisfy these
equations
( x -2 )( )
x -6 = 0

⎛ 81 119 ⎞ ⎛ 81 119 ⎞ ⎛ 81 119 ⎞ x = 2 (select), x = 6 (reject)


⎜ 8 , 4 ⎟ , ⎜ 8 , - 8 ⎟ , ⎜- 8 , 8 ⎟ x = 4
⎝ ⎠ ⎝ ⎠ ⎝ ⎠ So S contains exactly two elements 4 and 16.
⎛ 81 119 ⎞ 34. At least one digit (7) = Total numbers with 3 digit  –
and ⎜- 8 , - 4 ⎟ numbers which do not have
⎝ ⎠
So in A∩B, there are four points any digit as a ‘7’
= 9 × 10 × 10 – 8 × 9 × 9
31. M P = 900 – 648 = 252
a b c 35. Possible choices for a = 2(4, 5)
d Possible choices for d = 2 (0, 5)
e f
g
Ordered pairs of (b, c) = 6 [(3, 4), (3, 5), (3, 6), (4, 5),
(4, 6), (5, 6)]
C So total numbers are = 2 × 6 × 2 = 24
By given information
d = 18 (i) 36. At least one even digit = Total numbers – All digits are
b + d = 30 (ii) odd
e + d = 28 (iii) = 900 – 5 × 5 × 5
d + f = 23 (iv) = 775
a + d + b + e = 100 (v) 37. The number will have ‘2’ as two of its digits out 7 so
b + d + c + f = 70 (vi) we can select two places out of 7 for digit 2 is 7C2
e + d + f + g = 40 (vii) ways.

M05 PMRO -Math Olympiad XXXX 00_Solution.indd 8 12-Oct-20 11:17:50 AM


Combinatorics    5.9

Remaining 5 places is filled by 1 or 3. VAN _ _ _ _ = 2! = 2


So possible ways to fill five places are 25. VARNU = 1! = 1
So total 7 digit number with all given conditions VARUN = 1! = 1
= 7C2 × 25 So rank of ‘VARUN’ = 24 + 24 + 24 + 24 + 2 + 1 + 1
  = 100
7×6
=
× 32 = 672 42. We have 4 colour strips and we want a six vertical step
2×1
flag such that no adjacent strips are the same colour.
3 8. Number of ways to select four consonants from seven So choices for first strip is 4
consonants = 7C4 Then choices for second strip is 3 (not same colour
Number of ways to select 2 vowels from four vowels with first)
= 4C2 Again choices for third strip is 3 (not same colour with
and now we can arrange 6 letter by 6! Way so total second) and its so on.
words = 7C4 × 4C2 × 6 ! So total ways = 4 × 3 × 3 × 3 × 3 × 3
 = 151200  = 12 × 81
39. If number is divisible by 5, then two possibilities for 43. D C
unit digit
E
[1] When unit digit is ‘5’
88761 =8×8×7×6×1 A B

5 ABCD is a square and point E is the intersection point
[8 choices for first digit (except 0, 5)] of AC and BD.
Means point A, E, C and points B, E, D are collinear.
[2] When unit digit is ‘0’ Means from these pairs we cant construct any triangles
1 8 7 6 1 = 8 × 7 × 6 (4 times) So total triangle = 5C3 - 2

5

0
 = 10 – 2 = 8
[but 5 move at any four places so in total number 44. Number is divisible by 3 if the sum of digit is divisible
we can multiply by 4] by 3.
Total numbers = 8 × 8 × 7 × 6 + 8 × 7 × 6 × 4 So 5-digit number is divisible by 3, if we left one
= 8 × 7 × 6(8 + 4) ­number (3/0) from all 6 digits.
= 8 × 7 × 3 × 2 × 12 Case 1: From digits 1, 2, 3, 4, 5
= 168 × 4! Total numbers = 5! = 120
= k × 4! Case 2: From digits 0, 1, 2, 4, 5
So k = 168 Total numbers = 4 × 4 × 3 × 2 × 1= 96
So total numbers = 120 + 96 = 216
40. Terminal digits are filled by 3 × 2 ways because dig-
its are not repeated and for remaining digits we have 45. Total numbers with all digits
5 × 4 × 3 × 2 ways = 9!
3 5 4 3 2 2 In these numbers, in half of numbers 1 appearing same
where to the left of 2 and in half of numbers 1 appear-
2/4/6 2/4/6
ing some where to the left of 2, similarly for 3 and 4
and 5 and 6
Total ways = 3 × 5 × 4 × 3 × 2 × 2 = 720
9!
41. Alphabetical order of letters of word ‘VARUN’ is A, N, So total numbers =
2! 2! 2!
R, U, V = 9 × 7!
Number of words start with
A _ _ _ _ = 4! = 24 46. There are 10 people and we have to distribute 5 dif-
N _ _ _ _ = 4! = 24 ferent books to them in which each person cannot
U _ _ _ _ = 4! = 24 get more than one book. It means we have to arrange
V _ _ _ _ = 4! = 24 those 5 books in 10 places.

M05 PMRO -Math Olympiad XXXX 00_Solution.indd 9 12-Oct-20 11:17:51 AM


5.10  Chapter 5

So therefore, 5 different books can be distributed 51. A B C


among 10 people in 10P5 way. 4 4 4
10P5 = 10C5 × 5! Selection of five questions from 12 question with at
least one question from each group can be possible in
47. Indian 5 m 5w
(1, 1, 3) or (1, 2, 3) ways.
American 2 m 5w
We categories this problem into three cases So total ways
Case 1: When all 10 men shake hands is equal to selec- = ⎡⎣4C1 × 4C1 × 4C3 ⎤⎦ × 3 + ⎡⎣4C1 × 4C2 × 4C2 ⎤⎦ × 3
tion of 2 male from 10 men = 10C2
= (4 × 4 × 4)3 + (4 × 6 × 6) × 3
Case 2: When all 10 women shake hands is equal to
= 192 + 423
selection of 2 female from 10 women = 10C2
= 624
Case 3: Now every Indian male shake hand with every
American female so total ways = 5 × 5 = 25 52. For descending order ⇒
Case 4: Every American male shake hand with every digits are 9, 8, 7, 6, 5, 4, 3, 2, 1, 0 (total 10 digits)
American woman except his wife. So So if we can select any 5 digits from these 10 digits
Total ways = 5 × 4= 20 than only one possible way to decreasing order of dig-
Total shake hands = 10C2 + 10C2 + 25 + 20 its. So selection of 5 digits from 10 digits is 10C5 = m.
= 45 + 45 + 45 [‘0’ is smallest digit so if we select ‘0’ then its already
= 135 gone in last]
For ascending order:
48. Case 1: When exactly 3 of the first five questions are
Again we have 10 digits, but if we select ‘0’ then it go
answered. So remaining 7 out of remaining 8
to the first position which is not possible.
questions are answered by students
So we can select 5 digit from remaining 9 digits.
Total ways = 5C3 × 8C7 = 80
So possible ways = 9C5 = n.
Case 2: When exactly 4 of the first five questions are
answered by students Now m – n = 10C5 - 9C5
Total ways = 5C4 × 8C6 = 140
10 × 9 × 8 × 7 × 6 9 × 8 × 7 × 6 × 5
Case 3: When exactly 5 of the first five questions are = -
answered by students 5× 4 × 3× 2×1 5× 4 × 3× 2×1
Total ways = 5C5 × 8C5 = 56 9 × 8 × 7 × 6 × (10 - 5)
=
Total ways = 80 + 140 + 56 = 276 5× 4 × 3× 2×1
Answer: (A) 9×8×7×6×5
=
49. Total numbers with n-digits by using digits 1and 2 5× 4 × 3× 2×1
= 2n. = 9C5
In these numbers we have 2 numbers with all same
53. First of all we select 4 pairs out of 5 pairs which can be
digits (111 … 1 and 22 … 2)
done in 5C4 ways.
So Total numbers with atleast one is 1 and 2 is
Now for every selected pair select any one shoe which
2n – 2 = 510
can be done in 2C1 × 2C1 × 2C1 × 2C1 ways.
2n = 512
( )
4
So n = 9 So total numbers of ways = 5C4 × 2C1
50. We have arrange y counters = 5 × 24 = 80
1, 2, 3, … y 54. A and B must be next each other so first of all we have
So there are x choices for the 1st counters because we formed a group with A and B so now 6 persons left
have x-different colours counters again for 2nd count- with 1 group of A and B.
ers we have x choices and so on. AB , C, D, E, F, G, H
So arrangements = xy So total ways to arrange 7 persons = 7! × 2.
Now we are not include arrangement consists of all [We can exchange the position of A and B so we mul-
counters of the same colour. We have x colours so x tiply it by 2]
arrangements are possible. So subtract them from xy. Now in these arrangements in half arrangements C,
So total arrangements = xy – x above the D and in half D, above the C.

M05 PMRO -Math Olympiad XXXX 00_Solution.indd 10 12-Oct-20 11:17:53 AM


Combinatorics    5.11

7 !× 2 59. B1 B2 B3 B4
So total arrangements = = 7! 2 2 2 3
2!
= 5040 Toys receive.
9!
55. The number of times the teacher visit the garden = Total distribution = × 3!
2! 2! 2! 3! × 3!
Selection of 5 students from 25 students = 25C5
Now we have 25 students so we can select one student 9!
  =
who visit the garden, so remaining 4 students s­elected (2 !)3 × 3 !
by 24 times so each students visit the garden 24C4 60. _ 1 _ 2 _ 3 _ 4 _ … _ (n – 3) _
times. We have (n – 3 + 1) places to sit 3 persons in
So the number of visits, the teacher makes to garden (n - 3 + 1)C3 ways
exceeds that of a kid by 25C5 - 24C4 Its similar to select 3 persons from n persons such that
25 × 24 × 23 × 22 × 21 24 × 23 × 22 × 21 no two of them are consecutive.
- So Pn = n - 2C3
5× 4 × 3× 2×1 4 × 3× 2×1
24 × 23 × 22 × 21 and Pn + 1 = n - 1C3
5× 4 × 3× 2×1
[25 - 5] So Pn + 1 – Pn = 15
= 24C5 n - 1C3 - n - 2C3 = 15

56. 7 is divided into three groups such in each group have (n - 1) (n - 2) (n - 3) - (n - 2) (n - 3) (n - 4)
= 15
atleast one and none of are equal is 1, 2, 4 3× 2×1 3× 2×1
7! (n – 2) (n – 3) [(n – 1) – (n – 4)] = 15 × 3 × 2
So possible ways to distribution = × 3!
1! 2 ! 4 ! (n – 2) (n – 3) = 6 × 5
So n=8
7×6×5
= ×6
2×1 61. 5 subjects can be arranged in 6 periods in 6 × 5 × 4 ×
= 630 3 × 2 ways
Any of the 5 subjects can be organized in the remain-
57. Distribution of 8 apples to 3 boys such that each boy ing period in 5C1 ways.
get at least 1 apple and at most 4 apple is (1, 3, 4), Two subjects are alike in each of the arrangement. So
(2, 3, 3), (2, 2, 4) we need to divide by 2!
So total distribution 6×5×4×3×2
8! 8! 3! 8! 3! So total arrangements = × 5C1
= × 3! + × + × 2!
1! 3 ! 4 ! 2! 3! 3! 2! 2! 2! 4! 2! = 1800
⎡ 8 × 7 × 6 × 5 × 4 8 × 7 × 6 × 5 × 3⎤ 62. Consider the two green bottles as one entity let’s call it
⎢⎣8 × 7 × 6 × 5 + 2×2
+
2×2 ⎥⎦
GG.
8×7×6×5+8×7×6×5+2×7×6×5×3 So now we have 14 units (1 green pair, remaining 5
= 4620 = k × 7P3 green bottles and 8 blue bottles)
Now place the 8 blue bottles in a row, such that be-
4620
k= = 22 tween each bottle has a gap before and after it.
7×6×5 _B_B_B_B_B_B_B_B_
58. In first four digits we have one time 1, one time 2 and Now we need to take the six green bottles and place
4! them in six of the nine gaps.
two time 3. So we can arrange these four digits by
2! 9C6 = 84 ways
ways.
Now fifth digit is arrange by 2 ways (4 or 5) 63. First we arrange 10 red balls in a row by 10! ways
Again we know that sum of sixth and seventh digit is 9, So now we have 11 places between these red balls
so total 10 pairs are possible for 6th and 9th digit which _R1_R2_ … _R9_R10_
are (9, 0), (8, 1), … (1, 8), (0, 9). So arrange 9 green balls in 11 gaps by 11C9 ways
4! So total ways = 10 ! × 11C9
So total numbers = × 2 × 10 = 240
2!

M05 PMRO -Math Olympiad XXXX 00_Solution.indd 11 12-Oct-20 11:17:55 AM


5.12  Chapter 5

64. First we distribute (m + n) guests in two groups of m  = 400 + 80 + 16 + 3


and n guests by  = 499
(m + n ) ! Again no. of ciphers in 1001!
m !n ! ⎡ 1001 ⎤ ⎡ 1001 ⎤ ⎡ 1001 ⎤ ⎡ 1001 ⎤
= ⎢ + + + +…
Now we can arrange m friends on first table T1 by ⎣ 5 ⎥⎦ ⎢⎣ 25 ⎥⎦ ⎢⎣ 125 ⎥⎦ ⎢⎣ 625 ⎥⎦
(m - 1) ! ways = 200 + 40 + 8 + 1
= 249
2 So number of ciphers in 2002C1001 = 499 – 249 – 249
[Here clockwise and anticlock wise have same arrange-
 = 1
ments so it divided by 2]

Now similarly n friends arrange on table T2 by


(n - 1) ! 68. Any combination of 3 vertices can be formed a ­triangle.
2 Number of combinations of 3 that can be formed from
ways n choices = nC3

So total ways =
(m + n ) ! × (m - n ) ! × (n - 1) ! But in these cases we also count the cases when
m !n ! 2 2 (i) Exactly 2 sides in common with the polygon so
total triangles with 2 common sides = n
=
( m + n )! (ii) Exactly 1 side in common with the polygon than
4mn number of triangles = n(n – 4)
65. 8 is divided into two ways (4, 4) and (5, 3) Thus the required triangle = nC3 - n - n (n - 4 ) = 30
In first case, first we can select 4 from 8 and then re- n (n - 1) (n - 2)
maining 4 from 4. – n – n(n – 4) = 30
In second case, first we can select 5 from 8 and then 3× 2×1
remaining 3 from 3. n (n - 1) (n - 2) - 6n - 6n 2 + 24n
Total ways = 8C4 × 4C4 + 8C5 × 3C3 = 30
6
= 70 + 56 n(n – 1) (n – 2) – 6n2 + 18n = 180
= 126 n[n2 – 3n + 2 – 6n + 18] = 180
66. Road Road
n(n2 – 9n + 20) = 180
business x business n(n – 4) (n – 5) = 9 × 5 × 4
75-x-y 45-x-z
India (80) World (150) So n=9
5
y z So polygon in nonagon

25-y-z 69. Number is divisible by 25, if last 2 digits are 00, 25, 50
and 75.
Road business Case 1: End with 25, so remaining digits are 0, 1, 3, 4.
Today (30)
3 3 2
Total students = 100
(75 – x – y + x + y + 5) + 45 – x – z + z
+ 25 – y – z = 100 2 5
  145 + 5 – (x + y + z) = 100 So total 3 × 3 × 2 = 18 numbers
           x + y + z = 50 Case 2: End with 50, so remaining digits are 1, 2, 3, 4
So exactly two magazines read by 50 persons
2002 ! 4 3 2
67. 2002C1001 =
1001!× 1001!
Number of ciphers (zero at the end) in 2002! = 5 0
­maximum power of 5 which divides 2002!
So total 4 × 3 × 2 = 24 numbers
So no. of ciphers in 2002!
So total numbers are = 18 + 24 = 42
⎡ 2002 ⎤ ⎡ 2002 ⎤ ⎡ 2002 ⎤ ⎡ 2002 ⎤
= ⎢ + + + +…
⎣ 5 ⎥⎦ ⎢⎣ 25 ⎥⎦ ⎢⎣ 125 ⎥⎦ ⎢⎣ 625 ⎥⎦ 70. Selecting 3 people out of ‘n’ sitting in a row if no two
of them are consecutive.

M05 PMRO -Math Olympiad XXXX 00_Solution.indd 12 12-Oct-20 11:17:57 AM


Combinatorics    5.13

Arrange 3 people between (n – 3) persons in a row 74. We have 4 different toys and 5 same marbles.
such no two of them from 3 are consecutive For 4 toys: 4 toys are distributed like this 2, 1, 1.
So first we can sitting (n – 3) person, then we have 4! 3!
(n – 3 + 1) = (n – 2) alternate places to sit 3 people. So total ways = ×
2 !1!1! 2 !
So total ways = ( n - 2 )C = Pn(i) 24 × 6
3 = = 36
In a circular arrangement: We can subtract the 4
some cases from equation (1) that is, in circular ar- For 5 marbles: First we distributed 1 – 1 marble to
rangement, when we sitted (n – 3) persons than first each child.
and last arrangements are same so we can subtract that Then we have 2 marbles. Now these two marbles is
case when two persons sit first and last end (in row arr.) give to 3 children such that each child receive none or
and third person sit at remaining (n – 4) places. more than none.
= 3 + 2 - 1C3-1 = 4C2 = 6
So Qn = n - 3C3 – n - 4C1
So total possible ways = 36 × 6
Now Pn – Qn = 6 = 216
So n - 3C3 - ⎡⎣n - 3C3 - n - 4C1 ⎤⎦ = 6
75. We can see that minimum number of matches required
n -4 = 6 to win for India = 5
n –C14 = 6
(When all matches win by India)
So n = 10 And maximum number of matches = 9
(When India win 5 matches and Pakistan win
71. When we have 4 different books, then first we select
4 ­matches)
four persons from 10 persons then arrange these 4
Case: 1 When no. of matches = 5
books
So total ways to win 5 matches by India = select
So m = 10C4 × 4 !
5 matches from all 5 matches = 5C5 = 1 [wwwww]
But if books are alike, then no need to arrange the
Case 2: When number of matches = 6
books
In this case India must be win the last match and win
So n = 10C4
4 matches from first five matches.
m = n × 4! W
m = 24n So total ways = 5C4 = 5
72. We know 9 + 9 + 9 + 9 + 9 + 9 + 9 = 63 so if the Case 3: When number of matches = 7
sum of digits of 7 digit number is 61 then there are two In this case India must be win the last match and win 4
cases: match from first 6 matches.
[1] 5 times 9 and 2 times 8 W
[2] 6 times 9 and 1 time 7 So total ways = 6C4 = 15
So total possible numbers of 7 digit Similarly in case 4, case 5
7! 7! So total ways = 5C5 + 5C4 + 6C4 + 7C4 + 8C4
= + = 1 + 5 + 15 + 35 + 70
5 ! 2 ! 6 !1!
= 126
= 21 + 7 = 28
76. 1 2 3 4 5 6 7 8 . . . . . . . 99, 100
73. Number of ways that balls are arranged in a row so that
at least at least one ball is separated from the balls of
same colour. B1 2 B2 B3
= Total number of ways – All identical balls are a b c d
­together.
9! Suppose we can select 3 books B1, B2 and B3 from 100
= = 3! 4! [3 for 3 group and 4 for
books (1 to 100) such that
 2!3! 4 different green balls]
9 × 8 × 7! There are ‘a’ books above the B1, ‘b’ are books between
= - 3!4 ! B1 and B2 ‘c’ are books between B2 and B3 and ‘d’ books
2×6
= 6 × 7! – 6 × 4! below the B3 and a + b + c + d = 97 where a ≥ 0, b,
= 6(7! – 4!) c ≥ 1, d ≥ 0.

M05 PMRO -Math Olympiad XXXX 00_Solution.indd 13 12-Oct-20 11:17:59 AM


5.14  Chapter 5

So lets take another variable b′ and c′ such that b′ = [24 → because now we have 2 choices for each c­ ounter]
b – 1 and c′ = c – 1 [Subtraction of 2 for when all four counters have same
So b′ ≥ 0 and c′ ≥ 0 colour]
Put the values of b and c in equation So n = 81 – 3(14) –3
a + (b′ + 1) + (c′ + 1) + (d) = 97 = 36
a + b′ + c′ + d = 95 m 78 13
So = =
so Total possible ways = 95 + 4 - 1C4-1 n 36 6
= 98C3 6m = 13n
77. (x1 + x2 + x3) (y1 + y2) = 77 79. Let we have x books of algebra so remaining (12 – x)
 = 11 × 7 books of calculus
 = 7 × 11 Now possible number of ways to select 3 book of each
For positive integral values of (x1, x2, x3) and (y1, y2) topic is xC3 × (12 - x )C
we can factories 77 into two cases 11 × 7 and 7 × 11 3

So case 1: When x1 + x2 + x3 = 11 and y1 + y2 = 7 x (x - 1) (x - 2) (12 - x ) (11 - x ) (10 - x )


×
But here x1, x2, x3, y1, y2 ≥ 1 3× 2×1 3× 2×1
So take another variables So for the greatest value we can select x = 6
x11 = x1 – 1, x12 = x2 – 1, x13 = x3 – 1, y11 = y1 – 1 and So total books of algebra = 6 and books of calculus is
y12 = y2 – 1 also 6.
Such that x1, x2, x3, y1, y2 ≥ 0 (6 and 6)
So (x1′ + 1) + (y1′ + 1) + (z1′ + 1) = 11 80. (p + q) books distributed between two students x and
and (y1′ + 1) (y2′ + 1) =7 y such that x get p books and y gets q books
x1′ + y1′ + z1′ = 8 and y1′ + y2′ = 5 =
( p + q)! =L
so Total ways = 8 + 3 - 1C3-1 × 5 + 2 - 1C2-1 p !× q !
= 10C2 × 6C1 This case is similar to N, because we divided (p + q)
= 45 × 6 = 270 books in two groups of p and q. So
Case 2: When x1 + x2 + x3 = 7 and y1 + y2 = 11
L=N=
( p + q)!
By similar process p! q!
x1′ + x 2′ + x3′ = 4 and y1′ + y2′ = 9
Total ways = 4 + 3 - 1C3-1 × 9 + 2 - 1C2-1 But for the case of M, we have two choices that is x
    = 6C2 × 10C1 gets p books and y gets q books or x gets q books and
    = 150 y gets p books.
Total case = 270 + 150 So total ways are doubled.
    = 420
M=
( p + q)! × 2
78. m = The number of arrangements of four counts if no p! × q!
arrangement consists of counters of same colour So that  2L = M = 2N
  = Total arrangements when all four counters have 81. We have odd number of A so that middle term must be
same colour A and 2A’s and 3B’s are either side of this middle term.
So we have see that we have 3 choices for each count- A
ers so 2A’s and 3B’s 2A’s and 3B’s
m = 34 – 3 = 78 If we arrange the left side part of middle A then right
Now for ‘n’: side is already adjusted according to left.
n is the number of arrangement of four counters when So arrangement of 2A’s and 3B’s are
every arrangement consists of all 3 colour. 5!
So = 10
2!3!
n = Total arrangement – [arrangements with 2 colours]
– [
arrangements with only 82. Total ways to letters can be placed in envelops
1 colours] = 5! = 120 (without any restriction)
  = 34 – 3C2 [24 – 2] – 3

M05 PMRO -Math Olympiad XXXX 00_Solution.indd 14 12-Oct-20 11:18:01 AM


Combinatorics    5.15

But if we want at least two of them are in the wrong Now in statement – 2, we have 6A’s and 4B’s
envelops, then we can subtract that cases when exactly So number of ways to arranging them
one letter is in wrong envelope and all letters are in 10 !
correct envelope one letter is in wrong envelope is not   =
4 !6!
possible and there is only one way to all letters are in
10 × 9 × 8 × 7
correct envelops. So total cases = 120 – 1 = 119   = = 10C4
4 × 3× 2×1
83. (A) a1 < a2 < a3 … < an – 1 < an So statement – 1 is wrong and statement – 2 is correct
So number of increasing permutation of m symbols
are there from the n set numbers = nCm  (R) 87. Selection of novels by 6C4 ways and selection of dic-
(B) Every monkey have m choices for their masters and tionary by 3C1 ways.
we have n monkeys Now D
So total ways = m × m × m × … × m (n times) Dictionary always in the middle so we can arrange
    = mn(S) novels by 4! ways
(C) _ G _ G _ G _ G _ … _ G _ So total arrangements = 6C4 × 3C1 × 4 ! = 1080
We have ‘m’ gaps between (m – 1) green balls in- 88. Selection of two red balls from urn A = 3C2
cluding first and last gap so number of ways to se- Selection of two balls from urn B = 3C2
lecting n places from m So total ways = 3C2 × 9C2
= mCn  (Q)         = 3 × 36 = 108
(D) Every toy have n choices for child and we have m
toys so total ways 89. Statement – 1
= n × n × n × … × n (m times) First we put 1 ball in each box, then we have remaining
= nm (P) six balls.
So possible way to distributed 6 balls in 4 boxes
84. There are 10 candidate and 4, are to be elected and
= 6 + 4 - 1C4-1 ⎡⎣ by n + r - 1Cr -1 ⎤⎦
voter select atleast 1 candidate so possible ways
= 10C1 + 10C2 + 10C3 + 10C4 = 9C3
= 10 + 45 + 120 + 210 Statement – 2: Selection of 3 places from 9 different
= 385 places is also 9C3
85. S = {1, 2, 3, … 12} But statement – 2 is not a correct explanation for
Each set have equal size means all three sets A, B and ­statement – 1
C have 4 – 4 elements [answer correction]
Again there are no elements is common in all three sets 90. Triangle is formed by 3-non collinear points. So total
because A∩B = B∩C – C∩A = ϕ number of triangles = 10C3 - 6C3
So distribute 12 numbers in three groups which each    = 120 – 20 = 100
have 4 elements. Option (B) is correct
12 !
So number of ways = × 3! 91. Total ways = (10 + 1) (9 + 1) (7 + 1) – 1
4! 4! 4! 3!

12 ! When none of the
 =
(4 !)3 ball can be selected
= 11 × 10 × 8 – 1
86. Let a child take a1 icecreams of type – 1, a2 icecreams
= 879
of type – 2, a3 icecreams of type – 3, a4 icecreams of
type – 4, a5 icecreams of type – 5 92. Total elements in set A × B
So =4×2=8
a1 + a2 + a3 + a4 + a5 = 6 Now total subsets, of A × B having 3 or more elements
where a1, a2, a3, a4, a5 ≥ 0 = 8C3 + 8C4 + 8C5 + 8C6 + 8C7 + 8C8
So total possible ways are 6 + 5 - 1C5-1 = 10C4
[By using formula n + r - 1Cr -1 ] = 28 - ⎡⎣8C0 + 8C1 + 8C2 ⎤⎦
So statement – 1 is wrong = 256 – (1 + 8 + 28) = 219

M05 PMRO -Math Olympiad XXXX 00_Solution.indd 15 12-Oct-20 11:18:03 AM


5.16  Chapter 5

93. Number of triangles formed by joining vertices of an 5 4 3 2 1


n – sided regular polygon = nC3
So Tn = nC3 and Tn + 1 = n + 1C3 Total numbers = 5 × 4 × 3 × 2 × 1 = 120
Here Tn + 1 – Tn = 10 Total numbers = 72 + 120 = 192
n + 1C3 – nC3 = 10
5!
(n + 1) n (n - 1) n (n - 1) (n - 2 ) 97. Total words = = 60
- = 10 2!
3× 2×1 3× 2×1 Now alphabetical order of letters of the word ‘SMALL’
n(n – 1) [n + 1 – n + 2] = 60 is A, L, L, M, S.
n(n – 1) = 20 Now number of words beginning with
n(n – 1) = 5 × 4 4!
A – – – –  = = 12
So n=5 2!
L – – – – = 4! = 24
94.
D C 4!
(41, 41) M – – – –  = = 12
(0, 41) 2!
(1, 40)
(0, 40)
3!
SA – – –  = =3
2!
SL – – – – = 3! = 6
(40, 1) SMALL→1
(0, 1) So rank of ‘SMALL’ is = 12 + 24 + 12 + 3 + 6+ 1
A B
          = 58
(0, 0) (1, 0) (40, 0) (41, 0)
If we consider a square ABCD with side is 41 unit then 98. Let the female friends of man x represented by
we have the 40 possible values of x-coordinate and 40 M f1 , M f 2 , M f 3 , M f 4 and male friends of man x
possible values y-coordinate such that (x, y) lie in the represented by M m1 , M m2 , M m3 and male friends of
interior of the square. wife of man x is represented by Wm1 , Wm2 , Wm3 , Wm4
Means total 40 × 40 = 1600 points lie in the square. and female friends of wife of man x is represented by
In these points 40 point lie on the diagonal BD and W f1 , W f 2 , W f 3 .
half of remaining lie on either side of diagonals.
Now so possible cases of a party
So that number of points that lie in the triangle ABC
Case 1: 3M f / 3Wm = 4C3 × 4C3 = 16
1600 - 40
= = 780 Case 2: 2M f 1M m / 1W f 2Wm =
2
95. Total number of elements in set A × B = 4 × 2 = 8 4C2 × 3C1 × 3C1 × 4C2 = 324
Now total subsets of A × B each having at least three Case 3: 1M f 2M m / 2W f 1Wm =
elements = 8C3 + 8C4 + 8C5 + 8C6 + 8C7 + 8C8 4C1 × 3C2 × 3C2 × 4C1 = 144
    = 28 - ⎡⎣8C0 + 8C1 + 8C2 ⎤⎦ Case 4: 3M m / 3W f = 3C3 × 3C3 = 1
    = 256 – (1 + 8 + 28) So total cases = 16 + 324 + 144 + 1
    = 219        = 485
96. 4-digit numbers ⇒

3 4 3 2 Exercise – 1 Part – 2 KVPY level


problems
6/7/8
1. First of all we arrange the letters in alphabetical order
Total numbers = 3 × 4 × 3 × 2 = 72 A, G, K, N, R, U
5-digit numbers ⇒ Now number of words beginning with

M05 PMRO -Math Olympiad XXXX 00_Solution.indd 16 12-Oct-20 11:18:06 AM


Combinatorics    5.17

A – – – – – = 5! = 120 we choose P1 and P2 together or not and choose one


G – – – – – = 5! = 120 of them from P3 and P4 at a particular time.
KAG – – – = 3! = 6 Possible cases:
KANGRU  = 1! = 1 1. When P5 to P9 are selected = 5C5 = 1
KANGUR = 1! = 1 2. When P1 and P2 both selected and choose three
So rank of ‘KANGUR’ = 120 + 120 + 6 + 1 + 1 from P5 to P9 = 2C2 × 5C3 = 10
= 248 3. When P1 and P2 both selected, and one choose
from P3 and P4 and 2 choose from P5 to P9
2. If ‘5’ is one of digit of 3-digit number than it must be at
= 2C2 × 2C1 × 5C2 = 20
hundred’s place digit and ‘7’ must be at ten’s place digit.
4. When P1 and P2 both are not selected, one choose
1 1 5 from P3 and P4 and 4 choose from P5 to P9 =
2C1 × 5C4 = 10
So total cases = 1 + 10 + 20 + 10 = 41
5 7 Any even digit
6. A
So total numbers = 1 × 1 × 5 = 5
1 1
If ‘5’ is not a digit of 3-digit number then possible
2 2
choices of digit are
3 3
8 9 5

(Except 0 and 5) m n

So total numbers = 8 × 9 × 5 = 360 B C


So total numbers = 5 + 360 = 365 (i) When A is excluded the numbers of triangles
= nC2 × mC1 + mC2 × nC1
3. Let three digit is abc such that a > b, c
If a = 9 then possible values of b and c are form 0 to 8 n (n - 1) m (m - 1)
= ×m+ ×n
so total 9 × 9 numbers are possible. 2×1 2×1
If a = 8 then possible values of at a and b are from 0 to mn
7 so total 8 × 8 numbers are possible. =
2
[n - 1 + m - 1]
and So on
mn
So total numbers are = 92 + 82 + 72 + … 12 =
2
[ m + n - 2]
9 (9 + 1) (18 + 1) (ii) When ‘A’ is included: The one vertex is A one
    =
6 select from vertices of AB and one from AC. So
9 × 10 × 19 total triangles with included A = m × n
    = So now total triangle with or without
6
    = 285 mn
A = mn + (m + n - 2 )
4. There are two types of numbers possible abb and aab 2
here a ≠ 0 mn (m + n )
When number is in the form of abb then possible =
2
choices for digit is 9 × 9 × 1 = 81
mn (m + n - 2)
When number is in the form of aab then possible
choices for digit is 9 × 9 × 1 = 81 So required ratio = 2
So total numbers = 81 + 81 = 162 mn (m + n )
2
5. Let the persons P1 and P2 serve together or not and the
m+n -2
persons P3 and P4 refuse to serve with each other. So =
m+n

M05 PMRO -Math Olympiad XXXX 00_Solution.indd 17 12-Oct-20 11:18:08 AM


5.18  Chapter 5

7. Number of intersections points = selection of 2 lines Case 1: Select any three points from points 6 to 11
from 10 lines = 10C2 = 45 = 6C3 = 20
Now new fresh lines formed by these intersection Case 2: Select two points from points 6 to 11 and one
points = 45C2 - 10 × 9C2 points from 1 to 5
[But we can see that in each line 9 intersecting points = 6C2 × 5C1 = 75
are collinear 50 we can subtract 10 × 9C2 from total Case 3: Select one point from points 6 to 11 and two
cases] point from 1 to 5 = 6C1 × 5C2 = 60
45 × 44 10 × 9 × 8 Case 4: One circle from points 1 to 5
= – So total cases = 20 + 75 + 60 + 1
2 2
= 990 – 360 = 630       = 156
So total lines including old lines = 630 + 10 = 640 11. First we arrange the all sixteen players according to
8. Total squares = 6 × 4 + (6 – 1) (4 – 1) + (6 – 2) (4 – 2) there capabilities.
+ (6 – 3) (4 – 3) P1 > P2 > P3 … > P16
= 24 + 15 + 8 + 3 Now we formed four groups of 4 – 4 players but we
= 50 want P6 best player in one group so first we select P6
Total rectangles including square = Selection of 2 lines player in one group and then select 3 players from play-
from 5 horizontal lines × Selection of 2 lines from 7 ers P7 to P16.
vertical lines So total possible ways to from this group by 10C3 ways
= 5C2 × 7C2 Now we divided remaining 12 players into three
groups such that each group have 4 player is
5×4 7×6
= × = 210 12 !
2×1 2×1
Number of rectangles (not including squares) 4! 4! 4! × 3!
= 210 – 50 = 160 12 !
So total ways = × 10C3
9. Required number of ways = total numbers of ways (4 !)3 × 3 !
when A and B adjacent – total no. of ways when A 12 ! 10 × 9 × 8 12 !
and B adjacent with C and D adjacent. = × = 20 ×
When A and B are adjacent, then we formed a group (4 !) 3
6×6 (4 !)3
of A and B than arrange them by 5! × 2 ways So    k = 20
[we can exchange the A and B also, so multiplication 12. Case 1: When miss C is the member of committee
by 2] Subcase 1: When Mr. B is also the member of com-
When A, B and C, D are adjacent, then we formed mittee.
two groups of A and B and C and D. So total arrange- Then A not be member of the committee so selection
ments = 4! × 2 × 2 of 2 men from 4 men and 1 woman from women is
So required ways = 5! × 2 – 4! × 2 × 2 4C2 × 4C1 = 24 ways
 = 240 – 96 Subcase 2: When Mr. B is not a member of the
 = 144 committee then selection of 3 men from 6 men and 1
10. 6 7 women from 4 women is 5C3 × 4C1 = 40 ways
2
1 Case 2: When miss C is not the member of the com-
mittee then B is also not a member of the committee.
3 8 So selection of 3 men from 5 men and 2 women from
5 9 4 women is 5C3 × 4C2 = 60 ways.
4 10 So total ways = 24 + 40 + 60 = 124
11
We know that there is one and only one circle passing 13. 1000 = 23 × 53
through 3-non collinear points. All even factors is a multiple of minimum one power
So we can see that for a unique circle we need three of 2.
non-collinear points so total cases when circle is So product of all even divisions
formed.

M05 PMRO -Math Olympiad XXXX 00_Solution.indd 18 12-Oct-20 11:18:10 AM


Combinatorics    5.19

= (21 × 50) × (21 × 51) × (21 × 52) × (21 × 53) × (22 × So total selections = 6 + 4 + 6 + 6 + 3 = 25
50) × (22 × 51) × (22 × 52) × (22 × 53) × (23 × 50) × So answer is (c)
(23 × 51) × (23 × 52) × (23 × 53)
17. Number of ways in which at least one vowel is separat-
= 224 × 518
ed from rest of the vowels
= 26 × 218 × 518
= Total arrangements – When all vowels are together.
= 64 × 1018
11! 8!
14. We have total ‘n’ students, so first we select 25 students = -
4! 4! 2! 4! 2!
from n students and then arrange them.
So An = nC25 × 25! 8 ! ⎡ 11 × 10 × 9 ⎤
= ⎢ - 1⎥
4! 2! ⎣ 4 × 3 × 2 × 1 ⎦
Similarly An – 1 = n - 1C25 × 25 !
8 ! ⎡ 165 ⎤
Given that An – An – 1 = 25 ! × 49C25 = - 1⎥
4 ! 2 ! ⎢⎣ 4 ⎦
nC25 × 25 ! - n - 1C25 × 25 ! = 25 ! × 49C25
8 ! × 16 !
nC25 - n - 1C25 = 49C25 =
4 × 4! 2!
n!
-
(n - 1) ! = 49 ! 18. First we leave ‘S’ then remaining letters are P → 2, M
(n - 25) ! 25 ! (n - 26) ! 25 ! 24 ! 25 ! → 1, I → 4
7!
(n - 1) ! ⎡ n - 1⎤ = 49 ! No. of arrangements without S =
2! 4!
= 105.

( - 26) ! 25 ! ⎢⎣ n - 25 ⎥⎦ 24 ! 25 !
n In these 105 arrangement we have two types of ar-
(n - 1) ! =
49 ! rangement when
1. Both P are together:
(n - 26) ! 24 ! 24 ! 25 !
6!
So by comparison n = 50 No. of arrangements = = 30
4!
15. If no restriction to choose any discipline then we have [By grouping of both P]
total 3 × 3 × 3 × 3 × 3 × 3 ways to 2. When P’s are separated = 105 – 30 = 75
So total ways = 36. Now insert ‘S’ between letters:
But we want atleast one child is to be sent in each dis- Case 1: when ‘P’ are together: Then 1’S’ insert
cipline. So we can subtract these cases when all are in between both P and remaining S are in-
2 discipline and all are in 1 discipline. serting between gaps (including first a last
Number of cases when all are in 2 cases = 3C2 26 - 2 ( ) gap)
When all are in same discipline and total number of _ PSP _ M_ I _ I _ I _
cases when all are in 1 cases = 3 So total cases = 30 × 7C3 = 30 × 35 = 1050
So required possible ways = 36 - 3C2 26 - 2 - 3( ) Case 2: When ‘P’ are apart, then insert S between
   = 729 – 186 – 3 gaps of letters
   = 540 _P_M_P_I_I_I_I_
So total cases = 8C4 × 75 = 70 × 75 = 5250
Q (16 to 18)
MISSISSIPPI 10 !
So total cases = 1050 + 5250 = k ×
S → 4, I → 4, P → 2, M → 1 4! 4!
16. Selection of five letters from word ‘w’ is possible in 6300 × 4 ! × 4 !
k=
these ways 10 !
1. 4 alike 1 diff = 2C1 × 3C1 = 6 k=1
2. 3 alike 2 alike = 2C1 × 2C1 = 4 Q (19 to 20)
3. 3 alike 2 diff = 2C1 × 3C2 = 6
19. In each group we have 4 – 4 members so distribution
4. 2 alike 2 alike 1 diff = 3C2 × 2C1 = 6
of 16 players into four groups such that each group
5. 2 alike 3 diff = 3C1 × 3C3 = 3 have 4 players is

M05 PMRO -Math Olympiad XXXX 00_Solution.indd 19 12-Oct-20 11:18:14 AM


5.20  Chapter 5

16 ! So the four digit numbers with at least one digit from


= 0, 3, 6, 9 = 9000 – 1296
4! 4! 4! 4! 4!
= 7704
16 × 15 × 14 × 13 × 12 × 11 × 10 × 9 × 8 × 23. Consonants are C, R, C, M, F, R, N, C and vowels are
7× 6× 5× 4 × 3× 2×1 I, U, E, E, E
=
4 × 3 × 2 × 1× 4 × 3 × 2 × 1× 4 × 3 × 2 × 1× So selection of 3 consonants when
4 × 3 × 2 × 1× 4 × 3 × 2 × 1 (i) 3 alike = 1C1 = 1
(ii) 2 alike 1 diff = 2C1 × 4C1 = 8
15 × 13 × 11 × 9 × 7 × 5 × 3 × 7 (iii) All 3 are diff = 5C3 = 10 and selection of 3 vowels
=
3×3×3 when
35 (i) 3 alike = 1C1 = 1
= (13 × 11 × 9 × 5 × 7 × 3 × 1) (ii) 2 alike = 1C1 × 2C1 = 2
27
(iii) 3 diff = 3C3 =1
35 8 Now we have 9 cases of the words with 6 letters
= ∏ (2r - 1)
27 r =1 Case 1: From 3 alike consonants and 3 alike vowels
6!
20. First we select the players P1, P2, P3, and P4 in different numbers of words = 1 × 1 × = 20
groups then distribute 12 players into 4 groups such 3! 3!
12 ! 11! Case 2: From 3 alike consonants and 2 alike 1 diff
that each group have 3 players = =
3 ! 3 ! 3 ! 3 ! 4 ! 108 vowels
6!
21. Number is divisible by 3, if sum of digits is divisible Numbers of words = 1 × 2 × = 120
by 3. 3! 2!
1. Numbers with using the digits 0, 1, 2, 4, 8 Case 3: From 3 alike consonants and 3 diff vowels
4 × 4! = 96 6!
2. Numbers with using the digits 0, 1, 2, 7, 8 Numbers of words = 1 × 1 × = 120
3!
4 × 4! = 96 Case 4: From 2 alike 1 diff consonants and 3 alike
3. Numbers with using the digits 0, 1, 3, 4, 8 vowels
4 × 4! = 96 6!
= 8 ×1× = 480
4. Numbers with using the digits 0, 2, 4, 7, 8 2! 3!
4 × 4! = 96
Case 5: From 2 alike 1 diff consonants and 2 alike
5. Numbers with using the digits 1, 2, 3, 7, 8
vowels
5! = 120
6!
6. Numbers with using the digits 1, 2, 3, 4, 8 = 8×2× = 2880
5! = 120 2! 2!
7. Numbers with using the digits 2, 3, 4, 7, 8 Case 6: From 2 alike 1 diff consonants and 3 alike
5! = 120 vowels
So total numbers = 4 × 96 + 3 × 120 6!
= 8 ×1× = 2880
       = 384 + 360 2!
       = 744 Case 7: From 3 diff consonants and 3 alike vowels
22. Let the four digit number is abcd such that a, b, c and d 6!
= 10 × 1 × = 1200
are single digit numbers and a ≠ 0 3!
If number is divisible by 3 then one digit from a, b, c Case 8: From 3 diff consonants and 2 alike 1 diff
and d must be from 0, 3, 6, 9. ­vowels
Again total four digit numbers = 9 × 10 × 10 × 10 6!
= 10 × 2 × = 7200
= 9000 2!
Four digit numbers which have only digit 1, 2, 4, 5, Case 9: From 3 diff consonants and 3 alike vowels
7, 8 = 10 × 1 × 6 ! = 7200
= 6 × 6 × 6 × 6 = 1296

M05 PMRO -Math Olympiad XXXX 00_Solution.indd 20 12-Oct-20 11:18:16 AM


Combinatorics    5.21

So total words = 20 + 120 + 120 + 480 + 2880 + 2880 (v8, v9), (v9, v10)
+ 1200 + 7200 + 7200 Total 25 cases.
= 22100 And length is even in = 10C2 - 25
         = 20 cases
24. a × b × c = 2310
Similarly breadth is odd when we select the pairs of
a × b × c = 2 × 3 × 5 × 7 × 11
horizontal lines as mentioned below (H1, H2), (H1,
Case1: When none of number is 1.
H4), (H1, H6), (H1, H8) …
Then we can distribute 5 numbers {2, 3, 5, 7, 11} into
Total 20 cases and breadth is odd in 9C2 - 20
3 groups by two ways
= 16 cases
1. (1, 1, 3)
5! Now one side is even and one side is odd of rectangle
So total cases = = 10
3 !× 2 ! when we select odd length and even breadth or even
2. (1, 2, 3)
5! length and odd breadth.
So total cases = = 15 So total rectangle = 25 × 16 + 20 × 20
2! 2! 2!
         = 400 + 400
Case 2: When one numbers is ‘1’
         = 800
Then we can distribute 5 numbers {2, 3, 5, 7, 11} into
two groups by two ways. 26. When side length is 1 then sum of the area of squares
1. (1, 4)
5! is = (10 – 1) (9 – 1) (1)2 cm2
So total cases = =5 When side length is 3 then sum of the areas of squares
2. (2, 3) 4 ! is = (10 – 3) (9 – 3) (3)2 cm2 and soon.
5! So total area = (10 – 1) (9 – 1) (1)2 + (10 – 3) (9 – 3)
So total cases = = 10
2! 3! (3)2 + (10 – 5) (9 – 5) (5)2 + (10 – 7)
So finally we have total sets are = 10 + 15 + 5 + 10 (9 – 7) (7)2
= 40 4
∑ (11 - 2r ) (10 - 2r ) (2r - 1)
2
= cm2
Q Cp
(25(25-26)
– 26) r =1
H9 27. The first (non-zero) digit of the number ‘A’ can be any
one of nine (except O) then the second digit ‘B’ which
is used if there are two digits can be any one of the nine
digits (different from the first)
Now for remaining digits there are two digits used in
the number.
We have two possibilities A or B to fill each place but
we exclude ‘AAA’ as not involving two digits so there
are 23 – 1 = 7 possible pattern with exactly two differ-
H2
ent digits and 9 × 9 ways of choosing the pair of digits
in the first place.
H1
So total numbers with exactly two digits are 9 × 9 × 7
V1 V2 V3 V4 V10 = 557
Now when all digits are same then we have also 9 cases.
25. In rectangle we have two dimensions – length and So with not more than 2 different digits are have 557
breath + 9 = 576 numbers.
Now length is odd when we select the pairs of vertical
Q (28 – 30)
lines as mentioned below
Total number of ways they can be seated = 9! – 504(6!)
(v1, v2), (v1, v4), (v1, v6), (v1, v8), (v1, v10)
Let A, B and C represents the number of ways when
(v2, v3), (v2, v5), (v2, v7), (v2, v9)
2 American, 2 British and 2 Chinese sit together cor-
(v3, v4), (v3, v6), (v3, v8), (v3, v10)
responding.
(v4, v5), (v4, v7), (v4, v9)
A = 8 = C = 8! × 2!
(v5, v6), (v5, v8), (v5, v10)
(v6, v7), (v6, v9) 28. Now number of ways if exactly 2 pairs of same nation-
(v7, v8), (v7, v10) ality are together

M05 PMRO -Math Olympiad XXXX 00_Solution.indd 21 12-Oct-20 11:18:16 AM


5.22  Chapter 5

Case 2: When 3 are same flavour and 1 is diff like


3C2 [7 ! × 2 ! × 2 ! - 6 ! × 2 ! × 2 !] aaab/ abbb
↑ So first we select 2 flavour from 6 then possibilities for
 When all three pairs 3rd and 4th ice-cream is they have same flavour (a, a)
are sit together or (b, b)
3 × 6! (28 – 8) = 6C2 × 2 = 30
6! × 60 = P × 6! Case 3: When 2 are same flavour and 2 are diff same
So  P = 60 flavour like aabb.
= 6C2 × 2 = 30
29. Number of ways when American pair is adjacent (A) =
[2for (a, b) or (b, a)]
8! × 2!
Case 4: When 2 are same 1 diff and 1 also diff flavour
Number of ways when American pair and British pair
like aabc
are adjacent = 7! × 2! × 2!
= 6C3 × 3  [For last ice cream can be
Number of ways when American pair and Chinese
selected by 3 ways a/b/c]
pair are adjacent = 7! × 2! × 2!
= 60
Number of ways when all three pair are adjacent
So total cases = 6 + 30 + 30 + 60 = 126
= 6! × 2! × 2! × 2!
So required ways when only American pair is adjacent 34. If they contain 3 different flavours: first we select 3
= 8! × 2! – 7! × 2! × 2! – 7! × 2! × 2! + 6! × 2! × 2! × 2! flavour from 6, then for fourth ice-cream we have also
= 6! [112 – 28 – 28 + 8] 3 choices.
= 6! × 64 Ex → abca
= q × 6! So total ways = 6C3 × 3
So q = 64 = 60
30. Now number of ways in which atleast 2 people of same 35. If they contain only 2 flavour then first we select 2 fla-
nationality are together is vour from 6 then we have 3 choices for 3rd and 4th ice
A + B + C – [A∩B + B∩C + C∩A] – A∩B∩C creams
= 3(8! 21) – 3(7! 2! 2!) – 6! 2! 2! 2! Ex: If we select a, b flavour then 3rd and 4th ice-cream
= 6! (260) are selected from (a, b), (a, a) and (b, b)
Now required no. of ways when no two people of the So total ways = 6C2 × 3 = 45
same nationality are together If they contain only 3 flavour then we already find out
= 9! – 6! × 260 in previous question
= 6! × 244 = r × 6! So total ways = 60 + 45 = 105
So   r = 244
36. R + S + G + B = 15
31. nth term of an AP = a + (n – 1)d Now first we give 2 candy bar to shyam then we have
Let 361 is the nth of AP 13 candy bar left.
So 361 = 1 + (n – 1)k R + S1 + G + B = 13
(n – 1)k = 360 Here R, S1, G, B ≥ 0 but R ≤ 5
= 23 × 32 × 51 Case 1: If R = 0 then S1 + G + B = 13
Total number of factors of 360 = (3 + 1) (2 + 1) (1 + 1) Sot total cases = 13 + 3 - 1C3-1 = 15C2 = 105
   = 24
So k take 24 values Case 2: If R = 1 then S1 + G + B = 12
Sot total cases = 12 + 3 - 1C3-1 = 14C2 = 91
[Q 32 to 35]
Case 3: If R = 2 then S1 + G + B = 11
32. When all of different flavors then we can directly select Sot total cases = 11 + 3 - 1C3-1 = 13C2 = 78
4 ice-cream from 6 by 6C4 = 15 way Case 4: If R = 3 then S1 + G + B = 10
33. Possible pairs for ice-cream, if we consider we have 6 Sot total cases = 10 + 3 - 1C3-1 = 12C2 = 66
flavour a, b, c, d, e, f Case 5: If R = 4 then S1 + G + B = 9
Case 1: When all ice creams are same flavour Sot total cases = 9 + 3 - 1C3-1 = 11C2 = 55
= 6C1 = 6
Case 6: If R = 5 then S′ + G + B = 8

M05 PMRO -Math Olympiad XXXX 00_Solution.indd 22 12-Oct-20 11:18:18 AM


Combinatorics    5.23

 = 642 (remaining digits – 8, 0)


So total cases = 8 + 3 - 1C3-1 = 10C2 = 45  = 420 (remaining digits – 8, 6)
Finally total cases = 105 + 91 + 78 + 66 + 55 + 45 We have sum of remaining two digit is 8
= 440 So possible digits are 8 and 0.
37. Total persons = 8 and total seats = 9 Also know that A > B > C, so A = 8, B = 1 and C = 0
It means we have one vacant seat. If we select 8 seats So answer is 8.
from 9 than 1 vacant sit is also selected already. 72 ! 72 (72 - 8) (72 - 16)… (72 - 64 )
2. 8 =
So 9 is divided into three groups of 3 – 3: So first we 182 ! (18) (18 - 2) (18 - 4)… (18 - 16)
select 3 seats from 9 then again 3 seats from 6 then
again 3 from 3. 72 × 64 × 56 × 48 × 40 × 32 × 24 × 16 × 8
=
So total ways = 9C3 × 6C3 × 3C3 18 × 16 × 14 × 12 × 10 × 8 × 6 × 4 × 2
= (4)9
9×8×7 6×5×4 = 218 = 2x
= × ×1
3× 2×1 3× 2×1 So  x = 18
= 1680
3. First of all we can list out the ways that three periods
38. Case 1: when girls are in one car can be chosen for the mathematics courses when peri-
Car 1 3 G 3C3 = 1 ods cannot be consecutive (1, 3, 5), (1, 3, 6), (1, 4, 6),
(2, 4, 6)
Car 2 3 B 5C3 = 10
There are 4 ways to place 3 mathematics classes into 6
Car 3 2 B 2C2 = 1 periods for each of these we have 3! = 6 orderings of
So total cases = 1 × 10 × 1 × 3! = 60 the classes.
↑ So we have = 4 × 3! = 24 ways
for car Choose to classes.
Case 2: when 2 girls are in one car
4. Yoga Bridge
(a) Car 1 2 girls 3C2 = 3
Car 2 1G 2B 1C1 × 5C2 = 10 a b c
Car 3 3B 3C3 = 1 d
e f
So total cases = 3 × 10 × 1 × 3! = 180
(b) Car 1 2G 1B 3C2 × 5C1 = 15 g
Painting
Car 2 1G 2B 1C1 × 4C2 = 6 According to given conditions:
Car 3 2B 2C2 = 1 b + d + e + f = 9 (i)
So total cases = 15 × 6 × 1 × 3! = 540 a + b + d + e = 10 (ii)
(c) Car 1 2G 1B 3C2 × 5C1 = 15 b + c + d + f = 13 (iii)
Car 2 3B 4C3 = 4 d + e + f + g = 9 (iv)
a + b + c + d + e + f + g = 20  (v)
Car 3 1G 1B 1C1 × 1C1 = 1
Add equation (ii), (iii) and (iv)
Total cases = 15 × 4 × 1 × 3! = 360
(a + c + g) + 2(b + e + f) + 3d = 32
So finally total case = 60 + 180 + 540 + 360
From equation (v)
 = 1140
20 + b + e + f + 2d = 32
From equation (i)
Exercise – 1 Part – 3 KVPY level 20 + 9 + d = 32
d=3
problems So ‘3’ students taking all three classes.

1. GHIJ = 9753 (remaining digit – 1) 5. From left to right let the 5 seats be assigned to five
= 7531 (remaining digit – 9) friends the following values:
But A + B + C = 9; so one of digit must be 1. 1, 2, 3, 4, 5
So sum of remaining two digits is 8. Implication: A + B + C + D + E = 1 + 2 + 3 + 4 + 5
Now DEF = 864 (remaining digits – 2, 0) = 15

M05 PMRO -Math Olympiad XXXX 00_Solution.indd 23 12-Oct-20 11:18:20 AM


5.24  Chapter 5

After the friends change positions, it must remain true Now we have 125 numbers, in which the digit 1, 2, 3,
that A + B + C + D + E = 15. 125
Now Bhawner move 2 seats to the right chetna had 5 and 7 are equally appear. So each digit appear
5
moves 1 seat to the left Divya and Esha swap position. = 25 times at unit place, tens place and hundred place
So implication: also.
A + (B + 2) + (C – 1) + (D + 1) + (E – 1) So sum of all numbers
So for sum = 15, we have decrease the value of A by 1. = 102 × 25 (1 + 2 + 3 + 5 + 7) + 101
A→A–1 × 25 (1 + 2 + 3 + 5 + 7) + 25 (1 + 2 + 3 + 5 + 7)
To finish in an end seat, Asha must begin in seat 2, = 25 × 18 (111)
with the result that moving 1 seat to the left puts Asha = 49950 = ABC50
in seat 1. So A = 4, B = 9, C = 9
So correct answer is seat 2. Now A + B + C = 4 + 9 + 9 = 22
6. Five digit palindrome numbers are in the form of ‘abc- 9. In beginning knight at the unshaded squares after one
ba’ where a ≠ 0 and b and c are single digit numbers. more knight will be at a shaded square after one more
If we select first 3-digit of numbers then fourth and move knight at the unshaded square and so on.
fifth digit automatically sets so combination of first So after odd number of moves, the knight will at the
three digits shaded square and after even number of moves, the
= 9 × 10 × 10 =900 knight will be at the unshaded squares.
Also total palindrome numbers is 900 Also within 20 moves knight will be reach at any
900 = 10 × a square of chess board. But 20 is a even. So that after
a = 90 20th move, the knight will be at unshaded square and
7. Number of total ways without any restriction = 5 × 4 we have total 32 unshaded squares.
× 3 × 2 × 1 = 120 So after 20 consecutive moves knight will be reach at
Number of ways when ‘A’ run the first leg 32 squares.
10. First of all, the digit ‘0’ cant appear in the number if it
1 4 3 2 1 = 24
appeared it would have appear first, but then the num-
ber would not be an 11 digit number.
A So we have a non-decreasing sequence of digits d1, d2,
d3, …, dn where all numbers are from 1 to 9.
Number of ways when ‘D’ run the last leg
To specify such a sequence, it is sufficient to say where
1 2 3 4 1 = 24 the 1’s end and the 2’s start, where the 2’s end and the
3’s start and soon.
Now imagine we have 8 lines that we can place be-
D
tween the digits. The digits upto the first line are 1,
Number of ways when ‘A’ run the first kg and ‘D’ run the digits between the first and second lines are 2 and
the last leg soon.

1 3 2 1 1 =6 11. There are three type of numbers aabb, abab and abba
such that a ≠ b and a ≠ 0
Case 1: For aabb
A D
= 1000a + 100a + 10b + b
So required no. ways when ‘A’ can not run the first leg = 1100a + 11b
and D cannot run the last leg For 101 = 1111a – 11a + 11b
= 120 – [24 + 24 – 6] = m (101) – 11(a – b)
= 78 So number is multiple of 101 if
a – b = 0
8. Total 3-digit number when repetition is allowed a = b (but this not possible)
= 5 × 5 × 5 = 125

M05 PMRO -Math Olympiad XXXX 00_Solution.indd 24 12-Oct-20 11:18:21 AM


Combinatorics    5.25

So there are none of the number is multiple of 101 So total cases = 38


For 7: 1099a + a + 7b + 4b Finally total pairs of (m, n) = 13 + 13 + 38
= 7m + a + 4b = 64
So number is multiple of 7 if a + 4b is a multiple of 7.
13. We have two times 2, two times 3 and three times 5.
So possible ordered pair of (a, b) except (7, 7) are (1, 5),
5 is odd times so middle digit of 7-digit palindromes
(2, 3), (3, 1), (3, 8), (4, 6), (5, 4), (6, 2), (6, 9), (7, 0),
numbers must be 5 and 1-1 digit of 2, 3 and 5 is either
(8, 5), (9, 3).
side of the number.
Total = 11
So 11 numbers are multiple of 7. 5

 
Case 2: When number is in the form of abab. {2, 3, 5} {2, 3, 5}
abab = 1000a + 100b + 10a + b If we arrange first three digit numbers then automati-
= 1010a + 101b cally last three are arranged. So total arrangement of 3
For 101 ⇒ 1010 a + 101b is always multiple of 101 digits = 3! = 6.
So there are 9 × 9 ordered pair of (a, b) So total palindrome numbers are also 6.
So total numbers are 81.
14. Case 1: When Ajay sits on edge seat.
For 7 ⇒ 1008a + 2a + 98b + 3b
Then Bharat or Chandan can’t sit next to Ajay, that
= 7m + 2a + 3b
must mean either Deepak or Enees sits next to Ajay.
So number is multiple of 7 if (a, b) are (1, 4), (2, 1), (2,
After we pick either Deepak or Enees, then either
8), (3, 5), (4, 2), (4, 9), (5, 6), (6, 3), (7, 0), (8, 4), (9,
Bharat or Chandan must sit next to Deepak/Enees.
1), (9, 8). [except (7, 7)]
Then we can arrange the two remaining people in two
So there are 12 numbers of multiple of 7.
ways. Also there are two different edge seats that Ajay
So numbers which are multiple of 101 or 7 but not
can sit.
both are 81 – 12 = 69 numbers.
So total cases = 2 × 2 × 2 × 2 = 16
Case 3: When number is in the form of abba
Case 2: When Ajay does not sit in an edge seat.
abba = 1000a + 100b + 10b + a
In this case, then only two people that can sit next to
= 1001a + 110b
Ajay are Deepak and Enees, and there are two ways to
For 101 ⇒ m(101) – 9a + 9b
arrange them, and this also handles the restriction that
m(101) – 9(a – b)
Deepak can’t sit next to Enees. Then there are two
So number is multiple of 101 if a = b which is not pos-
ways to arrange Bharat and Chandan. However, there
sible so there is no number which is multiple of 101.
are three initial seats for Ajay.
For 7 ⇒ 7m + 5b
So total cases = 3 × 2 × 2 = 12
So number is multiple of 7 if b = 0/7.
So finally total arrangements = 16 + 12 = 28
So total numbers are 9 × 2 – 1 = 17
[except (7, 7)] 15. We have 81 multiples of 11 from 100 to 999.
So now finally required total numbers = 11 + 69 + 17 Now we know that, if some digits repeated twice, then
= 97 we make 3 permutations and if no repeated digits than
we have 6 permutations, but switching the hundreds
12. If m × n is divisible by 33 then m × n is by also divisible
and unit digits also get a multiple of 11.
by 3 and 11. So one number must multiple of 11.
So overall we have 3 permutations to each multiple.
Case 1: If one number from m and n is 1, 1 then other
So now we have 81 × 3 = 243 permutation. But we
number must be multiple of 3, and we have 13 multi-
have over counted some numbers which have a zero.
ple in the range of 1 to 40
So there are 110, 220, … 990have 9 extra permuta-
So total pairs = 13
tions and also there are 209, 308, … 902 have 8 extra
Case 2: IF one number is 22, then again other number
permutations.
must be multiple of 3, and we have 13 multiple in the
So finally we have 243 – 9 – 8 = 226 permutations
range of 1 to 40
So N = 226
So total pairs = 13
Case 3: If one numbers is 33, then other number is N -1 225
Then = = 45
any ,number is possible from 2 to 40 except 33 because 5 5
(m ≠ m)

M05 PMRO -Math Olympiad XXXX 00_Solution.indd 25 12-Oct-20 11:18:21 AM


5.26  Chapter 5

16. Total arrangement of 1, 2, 3, 4, 5 in a circle = 4! = 24 20. There are two types of numbers are possible aab and
Now we can see that if we choose one number at a abb such that a ≠ 0 and a ≠ b.
time we can always obtain subsets with sums 1, 2, 3, Case 1: “aab” type numbers
4 and 5. We can select the digit ‘a’ by 9 ways (except 0) and also
If we choose everything except 1, 2, 3 4 and 5 then we select the digit ‘b’ by 9 ways (except a) so total numbers
can obtain subsets with sums of 10, 11, 12, 13 and 14. = 9 × 9 = 81
This mean that we now only need to check for 6, 7, Case 2: “abb” type numbers
8 and 9. However once, we have found a set summing By the same reason, total numbers are 9 × 9 = 81
to 6, we can choose everything else and obtain a set N 162
summing to 9, and similarly for 7 and 8. Thus we only So N = 81 + 81 = 162 and = = 18
9 9
need to check each case for whether or not we can
21. There are 16 ways to choose the first square. So then
obtain 6 or 7.
3 remaining squares in its column and row chose to
We find that there are only 4 arrangements that satisfy
exclude 7 square from being chosen next time.
these conditions. But each of these is a reflection of
Now we have 9 ways to choose the second square. So
another. So we have total 2 arrangements
then 2 remaining squares in its column and row chose
17. Case 1: When both mathematics course algebra and to exclude 5 squares from being chosen next time and
geometry choosed → English is already choosed. So then we have 4 ways to choose the third square.
now we can select one course from remaining 3 course So the number of ways to choose 3 squares is 16 × 9 ×
so total ways = 3C1 = 3. 4, but the order in which you choose the square does
Case 2: When one of mathematics course is choosed not matter, so we divide it by 3!
⇒ 16 × 9 × 4
First we select one mathematics course from 2 cours- = = 96
3!
es then select 2 course from remaining 3 courses in [Change in answer, answer corrections]
2C1 × 3C1 = 6 ways
So total way = 3 + 6 = 9. 22. 28 people receive byes, so in the first round we have
only 72 players so 36 mates plays. So In the second
18. We know that factorials of larger numbers have a lot of round we have 28 + 36 = 64 people, so there are 32
zeros at the end matches. Now we have 32 people so there are 16
10! = 3628800 matches and soon.
11! = 39916800 So total matches = 36 + 32 + 16 + 8 + 4 + 2 + 1= 99
So we can not add the last two digit of 10 or more
than 10! 23. If a restaurant offers x type of main courses then restau-
Now we have only rant offers 2x type of appetizers. So select 1 dessert
1! + 2! + 3! + … + 9! from 3, 1 appetizer from 2x and 1 main course from x
= 1 + 2 + 6 + 24 + 120 + 720 + 5040 + 40320 + for the different combinations of dinner.
362880 We also know that year 2003 have 365 days. So we
= 409113 require the total combinations are greater than 365.
So sum of last two digits = 1 + 3 = 4 So 3C1 × xC1 × 2xC1 > 365
3x (2x) > 365
19. 7 prizes can be distributed in two ways (1, 1, 1, 1, 3) or
365
(1, 1, 1, 2, 2) according to given conditions. x2 >
So total different ways P 6
⎡ 7! 7! ⎤ x > 60.83
= ⎢ + ⎥ 5! So minimum integer value of x is 8.
⎣ 1! 1! 1! 1! 3 !× 4 ! 1! 1! 1! 2 ! 2 ! 3 ! 2 ! ⎦ AQ:
Mean restaurant offers at least 8 main courses. Pleas
[35 + 105] × 120 check
P = 16800 24. Let the number of students on the counsil be x. if thi
So for welcome committee we select two person from sente
P + 100 16900 130 x members by
Now = = = 65 is nee
2 2 2 xC2 = 10 here.

M05 PMRO -Math Olympiad XXXX 00_Solution.indd 26 12-Oct-20 11:18:23 AM


Combinatorics    5.27

x (x - 1) 28. We have one digit, two digit, three digit four digit and
= 20 five digit numbers from 1 to 9999.
2×1
Case 1: For single digit: We have only one number ‘8’
x(x – 1) = 5 × 4 whose sum of digit is 8.
So x=5 Case 2: For two digit: let the number is ‘ab’ such that
Now number of ways to select 3 person for planning a > 0 and a + b = 8
committee by 5C3 way Let another variable a′ = a – 1So a′ ≥ 0
5×4 a′ + 1 + b = 8
5C3 = = 10
2×1 a′ + b = 7
26. Let the ball is removed from the box is mth number ball [a′ ≥ 0 and b ≥ 0]
so. Total solution = 7 + 2 - 1C2-1 = 8C1 = 8
Sum of all number – m = 5048  ⎡ By n + r - 1C formula ⎤
n (n + 1) ⎣ r -1 ⎦
– m = 5048 Case 3: For three digit numbers: Let the numbers is
2 ‘abc’
n(n + 1) – 2m = 10096 Such that a+b+c=8
We can see that n(n + 1) is a product of two consec- Similarly a1 + b + c = 7
utive numbers so we find nearest number to 10096 Total solutions = 7 + 3 - 1C3-1 = 9C2 = 36
which is product of two consecutive numbers. Case 4: For 4-digit numbers
So 100 × 101 = 10100 (close to 10096) a + b + c + d = 8
101 × 102 = 10302 So a1 + b + c + d = 7
99 × 100 = 9900 Total solution = 7 + 4 - 1C4-1 = 10C3 = 120
If we take n = 99 then we get Case 5: For 5-digit numbers
9900 – 2m = 10096 a + b + c + d + e = 8
2m = –196 So a′ + b + c + d + e = 7
m = –98 (negative value of Total solution = 7 + 5 - 1C5-1 = 11C4 = 330
 m is not possible) Total numbers (n) = 1 + 8 + 36 + 120 + 330
If n = 100 then  = 495
10100 – 2m = 10096 Now 5(5w – n) = 5(500 – 495) = 25
m = 2 (possible)
If n = 101 then 29. Given that 0 < m < n < 2008 and
10302 – 2m = 10096 20082 + m2 = 20072 + n2
2m = 206 n2 – m2 = 20082 – 20072
m = 103 (but m < n) (n + m) (n – m) = (2008 + 2007) (2008 – 2007)
 so not possible (n + m) (n – m) = 4015 × 1
So only m = 2 is possible = 803 × 5
= 365 × 11
27. Observe that the give number has 2345 repeated 200 = 73 × 55
times and 2 + 3 + 4 + 5 = 14 so sum of all digits of [Also note that (n + m) > (n – m) so reject remaining
number is = 14 × 200 = 2800 cases]
So sum of first ‘m’ and last ‘n’ digits is 2800 – 2345 = So there are 4 cases for (m + n) and (n – m) but if we
455 and 455 = 32 × 14 + 7. Thus we have to cross out take 1st case then we get n = 2008 and it’s not possible
32 blocks of 4 digits 2345 either from the front or the because (n < 2008) so we have only 3 values of (m, n)
back and a 2 from the front that remains and a 5 from
AQ: the block that remains. Thus m + n = 32 × 4 + 2 = 130 30. First we remove all perfect square number tiles from 1
Please to 100 tiles so we removed 10 tiles. So remaining tiles
check m+n 130 after the first operation.
if this
Now = = 13
10 10 = 100 – 10 = 90
sentence
is needed [correction in Q 0 → remove, (m + n) → remove and Now we have 9 perfect square numbers from 1 to 90 so
here. find the] we removed 9 tiles then remaining tiles = 90 – 9 = 81.

M05 PMRO -Math Olympiad XXXX 00_Solution.indd 27 12-Oct-20 11:18:24 AM


5.28  Chapter 5

Again we have 9 perfect square numbers from 1 to 81, We want largest number so subtract the smallest sum of
so we removed 9 tiles then remaining tiles = 81 – 9 three numbers equation (iv) from equation (v)
= 72. (w + x + y + z) – (x + y + z) = 269 – 180
So the sequence of remaining tiles is 90, 81, 72, 64, 56, w = 89
49, 42, 36, 30, 25, 20, 16, 12, 9, 6, 4, 2, 1. So largest number is 89.
So total 18 times we performed to the operations.
34. First we arrange the six students on the chairs
31. Let x be the sum of the integers and y be the number S1 – S2 – S3 – S4 – S5 – S6
of elements in the list. Then according to questions. Now we have 5 gaps between two students for pro-
x + 15 x fessors.
= +2 So select 3 places from these five gaps and than arrange
y +1 y
3 professors.
xy + 15y = xy + 2y2 + x + 2y So total ways = 5C3 × 3 !
13y = 2y2 + x(1)  = 60
x + 15 + 1 x
Again = + 2 -1 35. To minimize the number of distinct values, we want to
y +1+1 y maximize the number of timer they appear.
x + 16 x+y Here 2018 = 223 × 9 + 10 +1
= So we have 23 numbers appear 9 times 1 number ap-
y+2 y
pear 10 times and 1 number appear 1 time.
xy + 16y = xy + y2 + 2x + 2y So total distinct numbers = 223 + 1 + 1
AQ: 14y = y2 + 2x(2) N = 225
Please Subtract two times of equation (1) from equation (2)
check N 225
if this
–12y = –3y2 So = = 45
So y = 4 (0 is reject) 5 2
sentence
is needed So there are 4 numbers in the original list. 36. A training zero requires a factor of two and a factor of
here. [Correction in Q add → when 1 app ended] five. Since factor of two occur more often than factor
of 5. So we can focus on the factors of 5. So we make
32. In Zolly we have last initial is Z so first two initials
a chart of zeros at the end of factorials have
from A to Y alphabets in alphabetical order.
So selection of 2 letters from 25 letters is 25C2 way Factorials 5! – 9! 10! – 14! 15! – 19! 20! – 24! 25! – 29!
and we have only one possible way to arrange them in Zeros 1 2 3 4 6
alphabetic order Case 1:
25 × 24 When n! has 1 zero and 2n! has 3 zeros
AQ: So total monograms (x) = 25C2 =
Please 2 For n = 8, 9 we have 1 zero but 16! and 18! Have 3
check x 300 zeros
if this So = = 10 Case 2:
sentence
30 30
When n! has 2 zeros and 2n! has 6 zeros.
is needed = 300
So for n = 13, 14 has 2 zeros but for 2n = 26 and 28
here. (Change the surname in question yadav → zolly)
has 6 zeros.
33. Let the four numbers are w, x, y, z such that w > x > y So smallest four values of n greater than 4 is 8, 9, 13 AQ:
Please
> z. and 14.
check
Now according to question Sum of least possible values of n = 8 + 9 + 13 + 14 if this
w + x + y = 222 (i) = 44 sentence
w + x + z = 208 (ii) Now sum of digits of 44 = 8 is needed
w + y + z = 197 (iii) [correction in Q → let ‘s’ and last → least] here.
x + y + z = 180 (iv)
37. If we have 1 as the first number then the only possible
Add all four equations, we get
list is (1, 2, 3, 4, 5, 6, 7, 8, 9, 10)
3(w + x + y + z) = 807
Now if we have 2 as the first number, then we have
w + x + y + z = 269  (v)
9 ways to choose where the 1 goes and the numbers
ascend from the first number 2 (except 1)

M05 PMRO -Math Olympiad XXXX 00_Solution.indd 28 12-Oct-20 11:18:26 AM


Combinatorics    5.29

For example: (2, 3, 4, 5, 1, 6, 7, 8, 9, 10) so there are and x = 1 + y(ii)


9C1 ways to do it. Now total score is 1 × x + 2 × y + 3 × z = 61
Now if we use 3 as the first number we need to choose Put the value of x and z
2 spaces to be 2 and 1 respectively. So there are 9C2 ⎛2 ⎞
ways. (1 + y) + 2y + 3 ⎜ y ⎟ = 61
⎝3 ⎠
By this similar way we have total ways 1 + 5y = 61
= 9C0 + 9C1 + 9C2 + … + 9C9 y = 12
By binomial theorem So x = 1 + 12 = 13 shots of 1 point shots
= 29 = 2k
So k = 9 40. We have 12 months in a year and 52 = 12 × 4 + 4
If we suppose in each month we have 4 people’s birth-
28. Let the ordered triple (a, b, c) denote that a songs are day then 4 people are left.
liked by anshul and yogesh, b songs by yogesh and sunil So by PHP rule we have at least four months in which
and c songs by sunil and anshul five people’s birthday.
We clam that the only possible triples are (1, 1, 1), So at least 5 people in the room have birthday falling
(2, 1, 1), (1, 2, 1), (1, 1, 2). in the same month.
To show this observe these are all valid conditions. Sec-
ond note that none of a, b, c can to bigger than 3. Sup- 41. We can see that there are 2 consonants between first
pose otherwise, that a = 3, without loss of generality two vowels A and E so we can partitioned by 3 way
say that anshul and yogesh like songs 1, 2 and 3. Then like.
because there is at least one song liked by each pair of A -T - H - E
↑ ↑ ↑
boys we require either b or c to be at least 1. In fact, we 1 2 3
require either b or c to equal 1, otherwise there will be Similarly there are 1 consonants between E and A so
a song liked by all three. Suppose b = 1 then we must number of way to partitioned is 2 and for last pair of
have c = 0 since no song is liked by all three boys, a vowels we have also 2 partitioned line. So
contradiction. Case 1:  When no partitioned so only one word
Case 1: For (a, b, c) = (1, 1, 1) “MATHEMATICS”
There are 4 choices for which song is liked by anshul Case 2: When two partitioned.
and yogesh, 3 choices for which song is liked by yo- (a) 3 vowels and 1 vowel ex: MATHEMAT-ICS
gesh and sunil, and 2 choices for which song is liked (b) 2 vowels and 2 vowels ex: MATHE-MATICS
by sunil and anshul. The fourth song can be liked by (c) 1 vowels and 3 vowels ex: MA-THEMATICS
only one of the boys or none of the boys, for a total of So total ways = 2 + 2 + 3 = 7
4 choices so Case 3: When three partitioned
(a, b, c) – (1, 1, 1) in 4 × 3 × 2 × 2 × 4 = 96 ways. (a) 2 vowels and 1 vowel and 1 vowel Ex: MATHE-
Case 2: For (a, b, c) = (2, 1, 1) MA-TICS
So there are 4C2 – 6 choices of songs for the first pair (b) 1 vowel, 2 vowels and 1 vowel Ex: MAT-HEMA­-
of boys. There remain 2 choices of song for the next TICS
pair (who only like one song). The last song is given to (c) 1 vowels, 1 vowels and 2 vowel Ex: MAT-HEM-
the last pair of boys. But observe that we let any three ATICS
pairs of the boys like two songs, so we multiple by 3. So total ways = 2 × 2 + 3 × 2 + 3 × 2 = 16
e
k
In this case there are 6 × 2 × 3 = 36 ways for the boys Case 4: When 4 partitioned:
s to like the songs (a) 1 vowels, 1 vowels, 1vowels and 1 vowel ⇒ ex:
nce So Total cases = 96 + 36 = 132 = 2k + 4 MA-THE-MAT-ICS
eded So  k = 7.
So total ways = 3 × 2 × 2 = 12
39. Let 1 point shots are x, 2 point shots are y and 3 points So finally total ways = 1 + 7 + 16 +12 = 36.
shots are z. 42. First select 4 hotels from 5, them arrange four visitors
Now according to question into hotels.
2y = 3z By 5C4 × 4 ! ways
2 = 5 × 24 = 120 ways
z = y(i)
3

M05 PMRO -Math Olympiad XXXX 00_Solution.indd 29 12-Oct-20 11:18:27 AM


5.30  Chapter 5

6 4
45. (i) Select any three digits from 6 numbers and then
43. (i) A ⎯⎯ → B ⎯⎯→ C arrange 6C3 × 3 ! = 120 numbers
So total ways from A to C via B
= 6 × 4 = 24 (ii) 2 5 4 = 40 numbers
(ii) There are 24 ways from A to C via B and again 24
ways from C to A via B. 2/3
So total ways = 24 × 24 = 576 (iii) 5 4 2 = 40 numbers
(iii) 5 A
6

B B 2/6
(iv) 5 4 4 = 80 numbers
3 4
C
First a person go A to B by 6 ways then B to C by 4 3/5/7/9
ways then back C to B by 3 ways (except one way
(v) 5 4 1 = 20 numbers
which is already used) then B to A by 5 ways. So
total ways.
= 6 × 4 × 3 × 5 = 360 ways 5
46. Odd digits are 1, 3, 5, 7, 9 so we have to choose 2 ­digits
44. Consonants → H, S, T, R, Y (5)
and then arrange by
Vowels → I, O (2)
5 4 = 20 numbers
(i) First we select 4 letters from 7 then arrange them
So total 20 numbers
by 7C4 × 4 ! = 840 ways.
(ii)  Select 4 consonants from 5 then arrange by 47. We have 4 digits 2, 3, 5 and 7 so we have 4 choices for
5C4 × 4 ! = 120 ways each digit of 7-digit telephone numbers so total tele-
(iii) First we select 2 consonants from 5 and then 2 phone numbers are
letters from remaining 5 letters = 4 × 4 × 4 × 4 × 4 × 4 × 4 = 47
So possible arrangements
48. (a) Each passenger have 3 choices for selections of rail-
5 5 4 4 = 400 ways
way carriage
P1 → 3, P2 → 3, P3 → 3, P4 → 3
Consonants Consonants So total ways = 3 × 3 × 3 × 3 = 34
(b) First we select 3 chairs from 4 for 3 persons and
(iv) 2 6 5 4 = 240 ways
then arrange
So total ways = 4C3 × 3 ! = 24
Vowel
49. First we select unit digits from 1 and 3 then select unit
(v) Select Y and 3 letters from remaining 3 letters digit from remaining digits (except 0) so total possible
and then arrange numbers are.
6C3 × 4 ! = 480 3 3 2 1 2 = 36 numbers

(vi) 1 5 4 2 = 40 ways
(Except 0 (1/3)
and unit digit)
T Vowel
(vii) 1
50. We have 7 colours and 7 places but green colour is
always in the middle so now we have 6 colours and 6
places so total arrangements are 6! = 720.
T
51 Total vowels = 3 (I, A, E)
Now select S and 2 more letters from remaining Total consonants = 4 (M, R, C, L)
5 letters and then arrange 5C2 × 3 ! = 60 So first we select 3 odd places from 4 odd places then
(viii) Select 2 consonants from 5 and 2 vowels from 2 arrange vowels then arrange consonants in remaining
vowels and then arrange places
5C2 × 2C2 × 4 ! = 240 ways 4C3 × 3 ! × 4 !

M05 PMRO -Math Olympiad XXXX 00_Solution.indd 30 12-Oct-20 11:18:29 AM


Combinatorics    5.31

52. Total possible attempts are 10 × 10 × 10 = 1000 56. Selection of 2 girls of first group and 2 girls of other 6
But in these attempts we have always 1 successful at- group from 4 girls are by 4C2 × 2C2 = 6 ways
tempt. Now first we arrange 5 boys by 5! ways. So there 6
So total unsuccessful attempts = 1000 – 1 gaps between boys (including first and last gap)
     = 999 B_B_B_B_B_
So selection of 2 places from 6 gaps by 6C2 ways
53. Total vowels = 4 (A, E, I, O)
Now we can also arrange the girls in a group by 2! ×
Total consonants = 7 (V, L, D, C, T, R, Y)
2! = 4 ways
So if we want vowels are always together than we
So total possible arrangements = 6 × 5 ! × 6C2 × 4
formed a group of 4 vowels. Now we have 8 letters
= 43200 ways
(7 consonants and 1 group of vowels) so possible ar-
rangements are 57. We can select the 8 question in these possible ways
= 8! × 4! [We can also arrange Part – A (7) Part – B (5)
 4 vowels in a group] (i) 3 5
= 967680 (ii) 4 4
(iii) 5 3
54. Let x1 = 3m1
So total possible ways
x2 = 3m2 + 1
= 7C3 × 5C5 + 7C4 × 5C4 + 7C5 × 5C3
x3 = 3m3 + 1
= 35 + 175 + 210
and we know x1 + x2 + x3 = 33
= 420
So 3m1 + 3m2 + 1+ 3m3 + 2 = 33
3(m1 + m2 + m3) = 30 58. First we select 1-1 horse from each of group of AA′,
m1 + m2 + m3 = 10 BB′ and CC′ by 2C1 × 2C1 × 2C1 = 8 ways
Such that m1, m2, m3 ≥ 0 Now we select 3 horses from remaining 10 horses.
So possible solutions of m1, m2, m3 Possible ways = 10C3 = 120
are (k) = 10 + 3 - 1C3-1  ⎡ ⎤ Total ways = 8 × 120 = 960
⎣ By n + r - 1Cr -1 Formula ⎦
 = 12C2 = 66 59. If 4th ticket bearing number 25, it means 3 tickets are
from 15 numbers (from 26 to 40) and 2 tickets are from
k 66 24 tickets (from 1 to 24)
So = =6
11 11 So possible ways = 24C2 × 15C3
55. Let there are x men participating in a chess tournament 50. Case 1: For 4 digit numbers:
So number of games between x men (a) When unit digit is ‘0’
= xC2 × 2  (Because every participant placed Total numbers = 9 8 7 1 = 504
 2 games with the other participant)
= x(x – 1) 0
Now number of games between men and women
(b) When unit digit is ‘5’
=2×2×x
= 4x Total numbers = 8 8 7 1 = 448
According to question
x(x – 1) = 66 + 4x 5
x2 – 5x – 66 = 0 Case 2: For three numbers:
(x – 11) (x + 6) = 0 (a) When unit digit is ‘0’
x = 11 or –6 [Negative value Total numbers = 9 8 1 = 72
of x is not
possible]
0
So x = 11
Total participants are = 11 + 2 = 13 (b) When unit digit is ‘5’
So total games = 2 × 13C2 Total numbers = 8 8 1 = 64
= 156
5

M05 PMRO -Math Olympiad XXXX 00_Solution.indd 31 12-Oct-20 11:18:32 AM


5.32  Chapter 5

Case 3: For two numbers: Case 4: When meeting point is D/F


(a) When unit digit is ‘0’ ⎡ 8! 8! ⎤
Total numbers = 9 1 =9
Total ways = 2 ⎢ × ⎥
⎣ 5! 3 ! 5! 3 ! ⎦
= 2 × 562
0
Case 5: When meeting point is E
(b) When unit digit is ‘5’ 8! 8!
Total numbers = 8 1 =8
Total ways = ×
4! 4! 4! 4!
  = 702
5 So total cases = 2(12 + 82 + 282 + 562) + 702
Case 4: For single digit number: only one number (5)  = 12870
So total cases = 504 + 448 + 72 + 64 + 9 + 8 + 1 62. First 2 clerks who prefer Bombay are to be sent in
 = 1106 companies of Bombay.
61. A I2 Now we select 2 clerks from 5 (excluding 3 clerks who
prefer for outside by 5C2 way.
B
Now we have 4 clerks for companies of Bombay
C so now we can select 2 clerks for first company and
D then select 2 more clerks from remaining 2 clerks by
4C2 × 2C2 ways
E
Now we have 6 clerks for other side companies. So
F first we select 2 clerks for first company and then 2
G more and then 2 more by 6C2 × 4C2 × 2C2 ways
So total possible ways = 5C1 × 4C2 × 2C2 × 6C2 × 4C2 × 2C2
H
      = 5400
63. Case 1: When teacher select 3 examples
I1 I
So we can select 1 – 1 examples from each by
Distance between I1 and I2 is 16 moves so both insect 4C1 × 5C1 × 6C1 ways = 120
meet after 8 – 8 moves. Also there meeting point is one Case 2: when teaches select 4 examples
of the from A, B, C, D, E, F, G, H and I. Now there are again three subcases.
Case 1: If meeting point is A/I ⇒ (a) A.P → 2, P and C → 1, B.T → 1
The number of ways the two insects can meet at point So total ways = 4C2 × 5C1 × 6C1 = 180
A or I are same by symmetry. Also the number of ways (b) A.P → 1, P and C → 2, B.T → 1
to reach at point A/I. From I1 and I2 are same. So total So total ways = 4C1 × 5C2 × 6C1 = 240
cases (c) A.P → 1, P and C → 1, B.T → 2
⎡ 8! 8!⎤ So total ways = 4C1 × 5C1 × 6C2 = 300
= 2 ⎢ × ⎥ = 2ways
⎣ 8! 8!⎦ Case 3: when teaches select 5 examples:
Now there are again 3 subcases
For A/I For I1 For I2 (a) A.P → 2, P and C → 2, B.T → 1
Case 2: IF meeting point is B/H: By same reasons Total ways = 4C2 × 5C2 × 6C1 = 360
⎡ 8! 8! ⎤ (b) A.P → 2, P and C → 1, B.T → 2
Number of ways = 2 ⎢ × ⎥ =2×8
2
So total ways = 4C2 × 5C1 × 6C2 = 450
⎣ 7 ! 1! 7 ! 1! ⎦
(c) A.P → 1, P and C → 2, B.T → 2
For B/H For I1 For I2 So total ways = 4C1 × 5C2 × 6C2 = 600
Case 3: When meeting point is C/G Case 4: When teaches select 6 examples
⎡ 8! 8! ⎤ A.P → 2, P and C → 2, B.T → 2
Total ways = 2 ⎢ × ⎥ Total ways = 4C2 × 5C2 × 6C2 = 900
⎣ 6! 2! 6! 2!⎦
= 2 × 282

M05 PMRO -Math Olympiad XXXX 00_Solution.indd 32 12-Oct-20 11:18:34 AM


Combinatorics    5.33

So total cases = 120 + 180 + 240 + 300 + 360 + 450 68. A never followed by B, means A followed by A or C.
+ 600 + 900 Similarly B followed by A or B and C followed by B
= 3150 ways and C.
Means each letter have 2 choices, that what is come af-
64. We can select First Square by 64 ways [any one from
ter that and we can start our sequence by 3 ways (from
64] but now we can see that 7 squares are present in his
A/B/C)
row and 7 squares are present in his column. So now
So if we have n letters than number of good words =
we have only (64 – 15 = 49) choices for 2nd squares.
3 × 2n
Total ways = 64 × 49
Here 3 × 2n = 384
But the order in which you choose the square does not
2n = 27
matter, so we divide it by 2!
So  n = 7
64 × 49
So total ways = = 1568 69. Vowels → U, I, E
2!
Consonants → M. L, T, P, L
65. First we select 2 passenger who insists on going up. Total arrangement of letters without any restriction =
Now we can select another 2 passengers from remain- 8!
ing 5 (exclude to refuse to go the upper deck) by 5C2 [2 for 2L’s]
2
way.
Now we arrange these four by 4! Way and then also ar- (i) We have 3 vowels so we can arrange by 3! = 6 ways.
range remaining 6 passenger in lower deck by 6! ways. We can see that in each way there is only one way
So total ways = 5C2 × 4 ! 6 ! = 172800 for same order of the vowels.
8!
66. Total number of ways to arrange all 8 letter is 8! So Total words = = 3360
2 × 3!
Total number of ways of arrangement of 4 vowels from
 But in these words ‘MULTIPLE’ word is also
4 vowels is 4! = 24.
count. So subtract 1 word from 3360.
But we can see that in these 24 arrangements there is
So Finally total ways = 3360 – 1 = 3359
only one arrangement in which all vowels are in alpha-
(ii) When each vowel is fixed in his original position
8!
betical order. So required ways = 5!
4! then we can arrange only consonants by ways.
2
8! 5!
= × 4! So total ways = -1 [1 for ‘MULTIPLE’]
4! 4! 2
= 59
= 8C4 × 4 ! 5!
(iii) Total arrangement of all five consonants by =
67. For finding the number of ways when two grandson 2
are never adjacent to each other we can subtract the 60 ways and total arrangements of all three vowels
number of ways when they are together from total by 3! = 6 ways
arrangements. So total arrangements = 60 × 6 – 1 [1 for
The position of grandfather is fix. So we can arrange “MULITPLE”]
other 6 members by 6! – 720 way.    = 359 ways.
Now if grandsons are together then they can be either 70. (i) First we select one person from 6 by 6C1 then sec-
side of grandfather. Also they can have two position ond person from remaining 5 persons by 5C1 and
in each side and we can also arrange by 2! Way. The then select third person from remaining 4 persons
granddaughters are also arrange by 4! Way. by 4C1 ways
So total arrangement = 2 × 2 × 2! × 4! So total arrangements = 6C1 × 5C1 × 4C1
    = 4 × 2 × 24 = 120 ways
    = 192 (ii) When there is no restriction then for each vacancy
So required ways = 720 – 192 we have 6 choices so total ways = 6 × 6 × 6 = 63
   = 528 = 216.

M05 PMRO -Math Olympiad XXXX 00_Solution.indd 33 12-Oct-20 11:18:35 AM


5.34  Chapter 5

(iii) In these 216 ways, we have 6 ways when 1st person Now when both K and U are together, by similar process
is selected by all 3 committee, 2nd person is select- 4!
A∩B = × 2 = 12
ed by all 3 committee 3rd person is selected by all 3 2!
and so on. Similarly B∩C = 12 = C∩A
So total required ways = 216 – 6= 210. Now when both K, U and T are together
71. IF number is greater than 600000 than 1st digit is great- A∩B∩C = 3! = 6
er than 5. So Number of ways when no two alike are together is
(a) When repetition is not allowed ⇒ = Total – [A + B + C – A∩B – B∩C – C∩B
Case 1: When 1st digit is even (6/8) + A∩B∩C]
= 90 – [30 + 30 + 30 – 12 – 12 – 12 + 6] = 30
2 4 3 2 1 3
73. We have 5 consonants D, G, M T, C and 3 vowels
O, A, I.
(6/8) (5/7/9)
So total arrangement of 5 consonants is 5! = 120
So total ways = 2 × 4 × 3 × 2 × 3 = 2144 and total arrangement of 3 vowels is 3! = 6 ways
Case 2: When 1st digit is odd (7/9) So Total number of words in which the relative order
2 4 3 2 1 2 of vowels and consonants remain unchanged = 120 ×
6 = 720
(7/9) (5, except unit digit)
But in 720 words “DOMATIC” is also include so re-
quired numbers = 720 – 1
Total ways = 2 × 4 × 3 × 2 × 1 × 2 = 96 = 719
So total numbers = 144 + 96 = 240 74. If 3a, 3b and 3c are in G.P than
(ii) When repetition are allowed. 3a × 3c = (3b)2
Case 1: When first digit is even (6/8) 3a + c = 32b
2 6 6 6 6 3 a + c = 2b
Means then a, b, c are in A.P.
(6/8) (5/7/9) So now we can select three terms from number 1 to
Total numbers = 2 × 6 × 6 × 6 × 6 × 3= 7776 101 so then they are in A.P
Case 2: When first digit is odd (7/9) Case 1: When common difference is 1 then we have
numbers (1, 2, 3), (2, 3, 4) … (99, 100, 101)
2 6 6 6 6 3
are in A.P. So total numbers = 99.
Case 2: When common difference is 2 then numbers
(7/9) (5/7/9) (1, 3, 5), (2, 4, 6) … (97, 99, 101) are in A.P.
Total numbers = 2 × 6 × 6 × 6 × 6 × 3 = 7776 So total numbers = 97 terms.
So Total numbers = 7776 + 7776 and so on
= 15552 So total cases are = 99 + 97 + 95 + … 3 + 1
= 502 [50 terms]
72. Let A represents the total no. of ways when both K are
= 2500
together B represents the total number of ways when
both U are together and C represents when both T are 75. (a) 21600 = 25 × 33 × 52
together. Total divisors = (5 + 1) (3 + 1) (2 + 1)
6! = 72
Total arrangements without restriction = = 90 If number N = P1x × P2y × P3z
2! 2! 2!
Then sum of all divisors =
For A: First we formed a group of both K then ar-
range all 5 letters. P1x +1 - 1 P2y +1 - 1 P3z +1 - 1
 × ×
(1 group of both K and remaining 4 letters) P1 - 1 P2 - 1 P3 - 1
5!
= = 30 26 - 1 34 - 1 53 - 1
2! 2! So sum of all divisors = × ×
2 -1 3 -1 5-1
Similarly B = C = A = 30 = (63) × (40) × 31 = 78120

M05 PMRO -Math Olympiad XXXX 00_Solution.indd 34 12-Oct-20 11:18:36 AM


Combinatorics    5.35

(b) 7056 = 24 × 72 × 32 77. B (9, 9)


Total divisors = 5 × 3 × 3 = 45
Let 2 factor are a and b such that a × b = 7056
So total pairs of (a, b) = 45 in which in one pair
a = b than in remaining pairs order of a and b are C (5, 5) N
interchanged
45 - 1 E
So Total ways = +1
2
 = 23
(c) 300 300 = 22 × 31 × 52 × 71 × 111 × 131
Let 300 300 = A × B A
We have 6 prime numbers (2, 3, 5, 7, 11, 13) so (0, 0)

for each prime number have 2 choices A and B. So If we find the number of paths that never pass through
total choices, (5, 5) = Total number of paths – Number of paths
2 × 2 × 2 × 2 × 2 × 2 = 64 when it passes through (5, 5)
But A × B = B × A so each pair is repeated two 18 !
Total ways from A to B =
times. 9 ! 9!
64
So total number of ways = = 32. [18 for total moves, 9 for east, 9 for north]
2
(d) 1010 = (2 × 5)10 = 210 × 510 = 48620
157 = 37 × 57 Total ways number of ways from A to C then C to B.
1811 = 211 × 322 10 ! 8!
= ×
Total factors of 1010 = 11 × 11 = 121 5! 5! 4 ! 4 !
Total factor of 157 = 2 × 8 = 64
Total factor of 1811 = 12 × 23 = 276 For A to C For C to B
HCF (1010, 157) = 57 252 × 70 = 17640
Total factors of 57 = 8 So required no. of ways = 48620 - 17640
HCF (1010, 811) = 210   = 30980
Total factor of 210 = 11
HCF (157, 1811) = 37 78. Let there are x books of algebra and (20 – x) books of
Total factor of 37 = 8 calculus.
And HCF (1010, 157, 184) = 1 So total ways to select 5 books from each
So number of positive integers that are divisor of at = xC5 × (20 - x )C
5
least one = Total individual factors – factors which
is common in two numbers + factor =
x ( x - 1) ( x - 2 ) ( x - 3 ) ( x - 4 ) ( 20 - x ) (19 - x ) (18 - x )(1
×
which is common in all. 1× 2 × 3 × 4 × 5 1× 2 × 3 × 4 ×
= (121 + 64 + 276) – (8 + 11 + 8) + 1
x ( x - 1) ( x - 2 ) ( x - 3 ) ( x - 4 ) ( 20 - x ) (19 - x ) (18 - x ) (17 - x ) (16 - x )
= 461 – 27 + 1 ×
= 462 – 27 = 435 1× 2 × 3 × 4 × 5 1× 2 × 3 × 4 × 5

76. B1, B2, B3, B4, B5, B6, B7, B8, B9, B10 ( x - 4 ) ( x - 3 ) ( x - 2 ) ( x - 1)( x ) (16 - x ) (17 - x )
Case 1: When we select two B consecutive books (18 - x ) (19 - x ) ( 20 - x )
from edges [B1B2/B9B10] then we have 7 =
5! 5!
choices for third book (except B3 or B8) So
This value is greater when x = 10
total cases = 2 × 7 = 14
So for greatest number of selections there are 10 books
Case 2: When we select two consecutive books from
on each topic in the library.
middle then we have 6 choices for third book
(except adjacent books on both sides)
So Total cases = 7 × 6 = 42

[For middle of 2 consecutive]


So Total ways = 14 + 42 = 56

M05 PMRO -Math Olympiad XXXX 00_Solution.indd 35 12-Oct-20 11:18:37 AM


5.36  Chapter 5

Exercise – 2 Part – 1 Previous 4. If we select any two lines from horizontal lines and any
two lines from verticals we get a rectangle.
year’s NSEJS Problems So n = 6C2 × 4C2 = 60
and m = 5C2 × 5C2 = 100
1. y 1 2 3 z So m>n+5
A B
5. A
First we divide the line AB up to YZ so we have three
parts (1), (2) and (3)
Now: U = (1) + (2) + (3)
P = (2) + (3) r
Q = (1) + (2)
R = (2)
So from option (A) B C
L.H.S = P∩Q = (2) 1
and R.H.S = R = (2) 3 2 3 Δ
So LHS = RHS
Area of equilateral Δ = (1) = and r =
4 4 s
Option (A) is correct
3 1
From option (B)    = =
3 2 3
LHS = P∩Q′ = ((2) + (3)) ∩ (3)) = (3) 4×
RHS = Q∩P = (2) 2
LHS ≠ RHS So area of circle = πr2
From option (C) 1
 = π ×
LHS = [P′∪Q′] + (1) + (3) 12
RHS = R = (2) So area of region inside the triangle but outside the
LHS ≠ RHS 3 π
From option (D) circle is = -
4 12
LHS = P∩Q′ = (3) So required probability
RHS = P′∩Q = (1)
Area inside the triangle but outside the circle
So LHS ≠ RHS =
So answer is (A) Area of triangle
[Correction in answer] ⎛ 3 π ⎞
[Also in option (B) → P∩Q′] ⎜⎜ - ⎟
⎝ 4 12 ⎟⎠ π
2. For n = 3 = = 1-
3 3 3
Total number of squares
1 × 1 size = 3 × 3 = 32 4
Total number of squares of 2 × 2 size 6. n(x) = 3
= 2 × 2 = 22 So the number of elements in power set of x = 23 = 8
Total number of squares of 3 × 3 size Again number of elements in power set of power set
=1×1 of x = 28
So total squares are = 32 + 22 + 12
7.
So for n order:
Total squares = n2 + (n – 1)2 + (n – 2)2 + … + 12
n (n + 1) (2n + 1)
AQ: =
Please 6
check 3. We have 14 lines, so if we select any two then we get a Number of squares in 4 × 3 grid
if this triangle. =4×3+3×2+2×1
sentence = 12 + 6 + 2
So total triangles = 14C2 = 91
is needed
here. [Change in question – diagram] there are 14 lines] = 20

M05 PMRO -Math Olympiad XXXX 00_Solution.indd 36 12-Oct-20 11:18:39 AM


Combinatorics    5.37

8. By the similar formula number of rectangles in m × n Part – 2 Previous years KVPY


grid.
Total number of cuboid
problems
m (m + 1) n (n + 1) p ( p + 1)
= × × 1. (1 + 2x)20 = a0 + a1x + a2x2 + … + a20x20
2 2 2 Put x = 1
5×6 5×6 5×6 320 = a0 + a1 + a2 + … + a20(i)
= × ×
2 2 2 Put x = –1
= 15 × 15 × 15 (–1)20 = a0 – a1 + a2 – … + a20(ii)
= 3375 Add (i) and (ii)
320 + 1 = 2(a0 + a2 – … + a20)
9. Let n1 and n2 are number of sides of polygons then
320 + 1
n2 (n2 - 3) So a0 + a2 + a4 + … + a20 = (iii)
n1 + = 103 2
2
2n1 + n22 - 3n2 = 206 (i) Now subtract (ii) from (i)
320 – 1 = 2(a1 + a3 + … + a19)
n1 (n1 - 3)
and + n2 = 80 320 - 1
2 So a1 + a3 + … + a19 = (iv)
2
n12 - 3n1 + 2n2 = 160 (ii) Now 3a0 + 2a1 + 3a2 + 2a3 + … + 2a19 + 3a20

Multiple equation (i) by 2 and (ii) by 3 and add = 3(a0 + a2 + … + a20) + 2(a1 + a3 + … + a19)
2n22 + 3n12 - 5n1 = 892 (iii) From equation (iii) and (iv)

Multiple equation (i) by 3 and (iii) by 2 and add ⎛ 320 + 1⎞ ⎛ 320 - 1⎞
= 3⎜ ⎟ + 2 ⎜ 2 ⎟
2n12 + 3n22 - 5n2 = 938 (iv) ⎝ 2 ⎠ ⎝ ⎠

Subtract equation (iii) from (iv)
5 × 320 + 1
n22 - n12 - 5n2 + 5n1 = 46 =
2
(n2 – n1) (n2 + n1 – 5) = 46
= 1 × 46 2. Total chances = 10 (from 1 to 10)
= 2 × 23 7th key is one chance from total 10 chances.
= 23 × 2 1
= 46 × 1 So required probability =
10
(n2 – n1) < (n2 + n1 – 5) so reject last cases.
Now case-1: When 3. Cricket Football
n2 – n1 = 1 and n2 + n1 –5 = 46 a p b
After solving we get x
q r
n2 = 26 and n1 = 25
But given condition of diagonal are not satisfy. c
Now case-2 when:
Tennis
n2 – n1 = 2 and n2 + n1 = 23 Let x% student like all three games.
after solving Now if x is minimum than p + q + r is maximum and
n1 = 13 and n2 = 15 a + b + c is minimum.
So total number of sides of two polygon is = 28 and so a+b+c=0
10. If sum is 7 then possible sums are (1 + 6), (2 + 5), p + q + x = 74 (i)
(3 + 4), (4 + 3), (5 + 2), (6 + 1) p + r + x = 76 (ii)
(Total = 6) q + r + x = 82 (iii)
and when sum is 11, then possible sums are (6 + 5), and p + q + r + x = 100
(5 + 6) (Total = 2) p + q + r = 100 – x(iv)
and total out comes = 6 × 6 = 36 Add equation (i), (ii) and (iii)
36 - 6 - 2 28 7 2(p + q + r) + 3x = 232
So required probability = = =
36 36 9

M05 PMRO -Math Olympiad XXXX 00_Solution.indd 37 12-Oct-20 11:18:41 AM


5.38  Chapter 5

From equation (iv) ⎡13 × 14 < 200 ⎤ AQ:


2(100 – x) + 3x = 232 m(m + 1) < 200 ⎢ ⎥ Pleas
 x = 32% ⎣14 × 15 > 200 ⎦ check
So maximum possible value of m = 13 if thi
4. First of all we formed five groups of number which are sente
Group (i) Multiple of 5 {5, 10, … 40} 8. If the number is divisible by 20 then last two digits are is nee
Group (ii) Multiple of 5 + 1 {1, 6, 11 … 36} 20, 40, 60, 80, 00. here.
Group (iii) Multiple of 5 + 2 {2, 7, … 37} → Ending with 00 are 9 numbers from
Group (iv) Multiple of 5 + 3 {3, 8, … 38} (100, 200, … 900)
Group (v) Multiple of 5 + 4 {4, 9, … 39} → If digits repeat other than zero than they are
If we want the sum of 2 numbers is not divisible by 5, (220, 420, 620, 820)
then we select a full group of numbers from group (ii) But each number can be permutated in two ways [Ex
or group (v). Also we can select a full group of num- → for 220 (220, 202)]
bers from group (iii) or group (iv). Also we can select a So there are 4 × 2 = 8 numbers
number from group 1 so numbers in subset of S. → If the number have no. digit repeated like (120) 120
=8+8+1 can be permutated in 4 ways like (120, 102, 210, 201)
So such numbers are 8 × 4 × 4 = 128
[Group (ii) or (v)] [group (iii) or (iv)] [one number So total numbers = 9 + 8 + 128 = 145
from group (i)] 145 29
So maximum numbers = 17 So required probability = =
900 180
5. 1
2 10 9. We have 3 numbers which are multiple of 9 7 numbers
which are multiple of 3 (except multiple of 9) and 20
3 9 numbers which are multiple of 1 (except multiple of 3
and 9)
4 8 So possible ways to choose numbers
1. All three are multiple of 9 ⇒ 3C1 = 3
5 7 2. Two are multiple of 9 and one is multiple of only 3
6 = 9C2 × 7C1 = 21
So if we join point 1 to 6, 2 to 7, 3 to 8, 4 to 9 and 5 3. Two are multiple of 9 and one is multiple of only 1
to 10, we get always 4 – 4 points on either side of line. = 9C2 × 20C1 = 60
So there are 5 lines. 4. One is multiple of 9 and two are multiple of only 3
= 3C1 × 7C2 = 63
6. First we arrange all vowels in 1 ways then we have 6 5. One is multiple of 9, one is multiple of only 3 and
gaps between two vowels (including first and last gap) one is multiple of 1
So we choose the 4 gaps from 6 gaps by 6C4 ways. = 3C1 × 7C1 × 20C1 = 420
_E_U_A_I_O_ 6. One is multiple of 9, and two are multiple of only 1
So total ways = 6C4 = 15 = 3C1 × 20C2 = 570
7. For maximum value of ‘m’, we start the numbers |A1|, 7. All three are multiple of only 3
|A2| … from minimum (from 1) such that A1, A2, … = 3 ⋅ 7C3 = 35
Am are distinct. 8. Two are multiple of only 8 and one is multiple of
So let |A1| = 1 only 1 = 7C2 × 20C1 = 420
 |A2| = 2 So total cases are = 3 + 21 + 60 + 63 + 420 + 570
 |A3| = 3 + 35 + 420 = 1590
   10. Such numbers are = from 11 to 19 → 9
 |Am| = m (22, 24, 26, 28) → 4
Such that A1, A2, A3 … are disjoint (33, 36, 39) → 3
But |A1| + |A2| + … + |Am| < 100 (44, 48) → 2
1 + 2 + 3 + … + m < 100 (55, 66, 77, 88, 99) → 5

M05 PMRO -Math Olympiad XXXX 00_Solution.indd 38 12-Oct-20 11:18:43 AM


Combinatorics    5.39

AQ: Total numbers (k) = 23 Case 2: (with repetition) (for m):


Please So k < 25 Subcase (a): When sequence is odd, even, odd even,
check [corrections in Q → write language → If k is the num- odd.
if this ber of elements in the set A then] So total cases = 3 × 2 × 3 × 2 × 3
sentence
= 108
is needed 11. n - 1C5 + n - 1C6 < nC7 
Subcase (b): When sequence is even, odd, even, odd,
here.  [we know that nCr + nCr -1 = n + 1Cr ]
even.
So nC6 < nC7 Total cases = 2 × 3 × 2 × 3 × 2 = 72
n! n! So Total cases (m) = 108 + 72 = 180
< m 180
6 ! (n - 6) ! (n - 7) ! 7 ! So = = 15
n 12
1 1
< 14. Let x1 < x2 < x3 … < x11
(n - 6) 7 So median is x6
7<n–6 Median of x1, x2 … x10 is
13 < n x3 + x6
So least value of n is 14 2
12. Equation of circle is So if we replace median by x11 then new numbers are
x2 + y2 = 1 x + x6
2 2
x1, x2, x3, x4, x5, 5 , x6, … x10
⎛ a⎞ ⎛ c⎞ 2
So ⎜⎝ ⎟⎠ + ⎜⎝ ⎟⎠ =1 x + x6 x + x6
b a So new median is 5 and 5 < x6 (old
­median) 2 2
a c2
= ± 1- 2 So median decreases
b d
a 1 2 2
= ± d -c
b d Part – 3 Previous Years PRMO
a Problems
is a rational number so
b
1. First we select 2 letters from 5 letters which is at the
d2 – c2 = perfect square (let k2)
correct address then de-arrange remaining 3 letters by
d2 = c2 + k2
d = even
2 ⎡ 1 1 1⎤
5C2 × 3 ! ⎢1 - + - ⎥ ways
c2 = odd ⎣ 1! 2 ! 3 ! ⎦
k2 = odd [∵ d and c are coprime] = 10 × (6 - 6 + 3 - 1)
d2 = (2p + 1)2 + (2q + 1)2 [Let c = 2p + 1 d = 2q + 1] = 20ways
d2 = 4[p2 + q2 + p + q] + 2
But every perfect square number is in the form of 4m, 2. There are 3 choices for the selection
4m + 1 (i) 1 red 1 green
Not in 4m + 2 (ii) 1 green 1 blue
So there is no value of S (iii) 1 blue 1 red
Means S is an empty set So total ways
= n - 1C1 × nC1 + nC1 × n + 1C1 + n + 1C1 n - 1C1
13. Even digit = 2 (2 and 4) 299 = n(n – 1) + n(n + 1) + (n + 1) (n – 1)
Odd digits = 3 (1, 3 and 5) 3n2 – 1 = 299
If sum of two adjacent digits is odd then one of them  n2 = 100
are even and other is odd. So n = 10
Case 1: (without repetitions) (For n):
There is only one sequence odd, even, odd, even, odd 3. Combination of (a, b) when a + b is divisible by 15 are
So possible numbers (n) = 3 × 2 × 2 × 1 × 1 = 12 (i) a = 15m + 1, b = 15m′ + 14
(ii) a = 15m + 2, b = 15m′ + 13

M05 PMRO -Math Olympiad XXXX 00_Solution.indd 39 12-Oct-20 11:18:44 AM


5.40  Chapter 5

(iii) a = 15m + 3, b = 15m′ + 12 2n + M = 13 (i)


(iv) a = 15m + 4, b = 15m′ + 11 and for S ∪ {15, 1}
(v) a = 15m + 5, b = 15m′ + 10 Sn + 15 + 1
(vi) a = 15m + 6, b = 15m′ + 9 =M+1
n+2
(vii) a = 15m + 7, b = 15m′ + 8 nm + 16 = (n + 2) (M + 1)
(viii) a = 15m, b = 15m′ nm + 16 = nm + n + 2M + 2
Here each pair represents each distinct colour. So there n + 2m = 14 (ii)
are maximum ‘8’ number of distinct colours possible. Subtract 2 times of equation (ii) from (i)
4. Let ‘x’ denote the set of all ordered pairs of (A, B) M – 4M = 13 – 28
when A ⊆ B and similarly let ‘y’ denote the set of all –3M = –15
ordered pairs (A, B) when B ⊆ A. M = 5
Here the number of subsets of {1, 2, 3, 4, 5} is 25. Put the value in equation (i)
Hence 25 ways to select set A and 25 ways to select B. 2n + 5 = 13
n = 4 AQ:
Since subsets A and B may be selected independently Please
by 25 × 25 = 210 ways. [Correction in q: M + 1 ↔ M + 2 check
Again A ⊆ B, then there are three possibilities for each and remove o4 from last] if this
of the five elements in the set {1, 2, 3, 4, 5}. Either it sentence
6. If we join any two vertices then we get a one line is needed
is in the set A set B – A, or nither subset. Thus there segment. here.
are 35 ordered pairs (A, B) with A ⊆ B. So number of ways to select 2 vertices from 8 by 8C2
= 28 ways
B
So there are 28 line segments.
7. Let there are ‘x’ women attended the party.
A
So total number of time the dance hold = 9 × 4 = x × 3
x = 12
So there are 12 women in the party
Similarly there are 35 ordered pair of (A, B) in which
8. First we classify the numbers in two parts from
B ⊆ A.
(i) 1 to 24 and (ii) 25 to 100
For the first condition when numbers are from 1 to 24,
A then there is no number available in the set such that
sum of two numbers is 125. So select all 24 numbers.
B Now for the second group, we formed pairs of the
numbers such that sum of numbers is 125. Pairs are
(25, 100), (26, 99), (27, 98) … (62, 63)
Now if A ⊆ B and B ⊆ A then A = B. Since each of So there are 38 pairs. If we select one number from
the five elements in {1, 2, 3, 4, 5} is either in A or each pair then sum 125 is not possible. So select 38
not in A. numbers from each 25 to 100 numbers.
So there are 25 ordered pairs of subsets (A, B) in which So total numbers = 24 + 38 = 62
A=B
So n(X′ ∩ Y′) = Total – [n(X) + n(Y) – n(X∩Y)] [From 1 to 24] [From 25 to 100]
 = 210 – 35 – 35 + 25 = 570 9. Let the ‘x’ fruit of first type ‘y’ fruit of second type and
5. Let there are ‘n’ elements in set S. So sum of all ele- ‘z’ fruit of third type can be purchased so
ments of S x + y + z = 25, [where x, y, z ≥ 0]
Sn = n × m(i) So possible solutions of (x, y, z) = 25 + 3 - 1C3-1 AQ:
Please
Now for S ∪ {15} ⎡ ⎤
S + 15
 ⎣ By n + r - 1Cr -1 formula ⎦ check
= n =M+2 = 27C2
if this
n +1 sentence
nm + 15 = nm + 2n + M + 2 = 351 is needed
[remove answer from question] here.

M05 PMRO -Math Olympiad XXXX 00_Solution.indd 40 12-Oct-20 11:18:45 AM


Combinatorics    5.41

10. B x(x – 1) y(y – 1) (y – 2) = 1200 × 6 × 2


A
  = 14400
C Here 14400 is product of 2 consecutive terms and 3
other consecutive terms.
E
D So possible factors of 14400 is = 4 × 3 × 2 × 25 × 24
There are 3-cases possible So by comparison
(i) A person visit 3 cities x = 25 and y = 4
(ii) A person visit 4 cities So total cups = 25 + 4 = 29
(iii) A person visit 5 cities
[Visit 2 cities is not possible] 14. E1 + 2E2 + 3E3 + … + 8E8
So for the first case, visitor select 2 cities from 4 except = (E1 + 4E4 + 7E7) + (2E2 + 5E5 + 7E5) + 3(E3 +
‘A’ by 4C2 and then arrange by 2! Way. 2E6)
Similarly for second and third, visitor select 3/4 cities = (E1 + E4 + E7) + 2(E2 + E5 + E5) + 3m
from 4 except A then B arrange them.  (m is a integer)
se So total cases = (E1 – E2) + (E4 – E5) + (E7 – E8) + 3q
k
4C2 × 2 ! + 4C3 × 3 ! + 4C4 × 4 ! (q is an integer)
is
12 + 24 + 24 = 60 ways Let x12 = E1 – E2
ence x45 = E4 – E5
eded
11. There are two ways to select the rooms x78 = E7 – E8
.
(i) 1 2 3 4 1 2 3 4 So we get x12 + x45 + x78 + 3q
G1 G3 G2 G4 ∵ E1, E2, … E8 ∈ {–1, +1}
So possible values of x12, x45 and x78 is 2/0/–2
G2 G4 G1 G3 For divisibility of 3 there are three cases.
Case 1: If x12 = x45 = x78 = 0
1 2 3 4 1 2 3 4
Total 2 × 2 × 2 = 8 cases
2 3 4 (ii) 1 2 3 4 Case 2: If x12 = x45 = x78 = 2/–2
G3 G2 G4 So total 2 cases.
Case 3: If x12 = 2, x45 = –2, x78 = 0
G2 G4 G1 G3 2 × 3! Cases = 12 cases
2 = [for x78]
2 3 4 1 2 3 4
3! = (by arranging all three)
So total arrangement = 2 × 4! So total cases = 8 + 2 + 12 = 22
= 48 ways Now we can also change E3 and E6 be any one from
12. 108 = 22 × 33 1 or –1.
Here a × b × c = 108 = 22 × 33 So total cases = 22 × 2 × 2 = 88
So let a = 2α1 × 3β1 So there are 88 types.
b = 2α 2 × 3β2 15. Sum of all numbers of the set
c = 2α 3 × 3β3 20 × 21
Such that α1 + α2 + α3 = 2 and β1 + β2 + β3 = 3 = = 210 = 2 × 3 × 5 × 7
2
By using formula n + r - 1Cr -1
So Number of partition Sum
Total solutions ⇒ 2 + 3 - 1C3-1 × 3 + 3 - 1C3-1 (i) 2 105
= 4C2 × 5C2 = 60 (ii) 3 70
(iii) 5 42
13. Let there are ‘x’ cups with handle and ‘y’ cups without (iv) 6 35
handle such that x, y ∈ N. (v) 7 30
se So number of ways to select two cups from y and 3 (vi) 10 21
k cups from x by Also see that partition is not greater than by 10 because
s yC2 × xC3 ways
ence we have only 20 numbers.
eded
So xC3 × yC′ 2 = 1200
.

M05 PMRO -Math Olympiad XXXX 00_Solution.indd 41 12-Oct-20 11:18:46 AM


5.42  Chapter 5

For case 1: Sum is 105 and number of partitions are 2 {4, 5, 6, 19, 20, 21, 34, 35, 36, 49, 50, 51, 64, 65, 66,
{1, 2, 3, 4, 5, 16, 17, 18, 19, 20} and {6, 7, 8, … 14} 79, 80, 81, 94, 95, 96, … 184, 185, 186}
So k = 105 possible So by observation 184 is the smallest number, when it
For Case 2: Sum is 70 and number of partitions are 3 is divided by 11, 13, 15 leaves remaining in {7, 8, 9},
{1, 2, 3, 4, 5, 6, 7, 8, 9, 11, 14}, {10, 12, 15, 16, 17} {1, 2, 3}, {4, 5, 6} respectively.
and {13, 18, 19, 20} Sum of squares of digits = 12 + 82 + 42 = 81
So k = 70 is possible.
18.
Case 3: For sum = 42 and partitions are 5
{20, 10, 12}, {18, 11, 13}, {16, 15, 9, 2}, {19, 8, 7, 5,
3} and {1, 4, 6, 14, 17}
So k = 42 is possible
Case 4: For sum = 35 and partitions are 6
{20, 15}, {19, 16}, {18, 17}, {14, 13, 8}, {12, 11,
10, 2}, {1, 3, 4, 5, 6, 7, 9}
So k = 35 is possible.
Case 5: For sum = 30 and partitions are 7
{20, 10}, {19, 11}, {18, 12}, {17, 13}, {16, 14}, {1,
5, 15}, {2, 3, 4, 6, 7, 8}
Let the chess board is like as mentioned above. In this
So k = 30 is possible
61 are shaded squares and 60are unshaded. So if we
Case 6: For sum = 21, and partitions are 10
want 60 unit squares from 121 unit square such that no
{1, 20}, {2, 19}, … {10, 11}
two chosen have a side in common. Then we select 60
So k = 21 is also possible
squares from 61 shaded squares or from 60 unshaded
So there are 6 god numbers
squares.
16. Let angles of triangles are x, y and z such that x + y + So total ways = 61C60 + 60C60
z = 180 (x, y, z ≥ 1) = 61 + 1
So x′ + y′ + z′ = 177 (x′, y′, z′ ≥ 0) = 62
Total solutions = 177 + 3 - 1C3-1 = 179 × 89 = 15753
20. There are two ways to partition 8 in unequal size
We have 3 choices of angles
which are
(i) When all three are equal (α, α, α)
(1, 2, 5) and (1, 3, 4)
(ii) When two are equal (α, α, β)
So Total ways to distribution (N)
(iii) When all are distinct (α, β, γ)
So (i)  Total triangle of type-1 is 1 ⎛ 8! 8! ⎞
= ⎜ + 8!
(ii) For case 2, 2α + β = 177 ⎝ 1! 2 ! 5 ! 1! 3 ! 4 !⎟⎠
 β must be odd so there are 88 solutions but we  = 2688
3! Sum of digit of N = 2 + 6 + 8 + 8 = 24
can also arrange by = 3 ways
2! 21. There are two ways for all five numbers
So total triangles = 88 × 3 = 264
1 1
15753 - 264 - 1
(iii) Total triangle of third type =
3! 3 4 4 3
 [We can arrange all angles by 3! Ways so for
unique triangles we can divide it by 3!]
= 2611
5 2 2 5
So finally total triangle of all three types
N = 2611 + 88 + 1 = 2700 and we can arrange by 5 ways to each.
N 2700 So total arrangements = 2 × 5 = 10
= = 27
100 100 22. Case 1: If we take gap of 2 sides then from figure
17. List of numbers, when divided by 15, gives in the we have 4C2 ways to select 2 parallel lines. Also we
­remainder {4, 5, 6} are can short from 1, 2, 3, 4 or 5 so total parallel lines =
5 × 4C2 = 30 ways.

M05 PMRO -Math Olympiad XXXX 00_Solution.indd 42 12-Oct-20 11:18:48 AM


Combinatorics    5.43

1 10 So total possible values of colouring = nCr × 2(n - r )


2 9 and r = 0, 2, 4, …
So total arrangement
n
3 8 f(n) = ∑ nCr × 2n - r
r = 0, 2, …
4 7
f(n) = nC0 2n + nC2 ⋅ 2n - 2 + nC4 2n - 4 … (1)
5 6 Now we know that
(2 + 1)n = nC0 2n + nC1 ⋅ 2n -1 + nC2 ⋅ 2n - 2 + … nCn (2)
Case 2: When we take gap of 3 sides then from figure
and
we have 3C2 = 3 ways to select 2 parallel lines.
(2 – 1)n = nC0 2n - nC1 ⋅ 2n -1 + nC2 ⋅ 2n - 2 - … ( -1) nCn
n

1 10 (3)
2 9 Add equation (2) and (3)
(
3n + 1 = 2 nC0 2n + nC2 ⋅ 2n - 2 + … )
3 8
From equation (1)
3n + 1 = 2f(n)
3n + 1
4 7 So f(n) =
2
5 6
39 + 1 33 + 1
So f(9) = and f(3) =
Also we can start from 1, 2, 3, 4 or 5. 2 2
So total ways = 5 × 3 = 15 f (9 ) 39 + 1
So total = 30 + 15 = 45 pair of parallel diagonals. = 3 = 36 – 33 + 1
f (3 ) 3 + 1

23. For the maximum prime factor one of the set A and set = 703 = 19 × 37
B contain only odd number. Largest prime factor is 37.
The possible arrangements of sets of A and B is
Cases Set A Set B Comment 25. Let us assume the 2A’s are A1 and A2
Case 1 5 6, 7, 8, 9 5 + 6 × 7 × 8 × 9 = 3029 So word is BHA1RA2T
divisible by 13 Total no. of arrangement of these 6
Case 2 7 5, 6, 8, 9 7 + 5 × 6 × 8 × 9 = 2167 ⎡ 1 1 1 1 1 1⎤
= 6 ! ⎢1 - + - + - + ⎥ = 265
divisible by 11 ⎣ 1! 2 ! 3 ! 4 ! 5! 6 ! ⎦
Case 3 9 5, 6, 7, 8 9 + 5 × 6 × 7 × 8 = 7209 Now P be the set when A2 occupies place of A1 and Q
divisible by 9 be the set when A1 occupies place of A2
Case 4 5, 7 6, 8, 9 5 × 7 + 6 × 8 × 9 = 467
265 265
which is prime So n(P) = = 5.3 and n(Q) = = 53
Case 5 5, 9 6, 7, 8 5 × 9 + 6 × 7 × 8 divisible 5 5
by 3 ⎡ 1 1 1 1⎤
and n(P∩Q) = 4 ! ⎢1 - + - + ⎥ = 9
Case 6 7, 9 5, 6, 8 7 × 9 + 5 × 6 × 8 divisible ⎣ 1 ! 2! 3! 4!⎦
by 3 (Dearrangement of remaining 4 letters)
Case 7 5, 7, 9 6, 8 5 × 7 × 9 + 6 × 8 divisible
So required arrangements
by 3
1
So largest prime number (N) is 467 sum of digits of = [265 - 53 - 53 + 9]
N = 17 2
 = 84
24. ....
⎡1 ⎤
⎢⎣ 2 because we have two A’s ⎥⎦
(1xn grid)
If we select r block (where r is even) for red colour then 26. {10 a + b : 1 ≤ a ≤ 5, 1 ≤ b ≤ 5}
remaining (n – r) blocks have 2 choices for the colour So we have total 25 numbers. These numbers are di-
blue or green. vided into 5 groups

M05 PMRO -Math Olympiad XXXX 00_Solution.indd 43 12-Oct-20 11:18:50 AM


5.44  Chapter 5

Group-1 = (11, 12, 13, 14, 15) 28. Set is {1, 2, 3, … n} this set can be partitioned into 3
Group-2 = (21, 22, 23, 24, 25) subsets with equal sums, so that sum is divisible by 3.
Group-3 = (31, 32, 33, 34, 35) n (n + 1)
Group-4 = (41, 42, 43, 44, 45) Means is divisible by 3.
2
Group-5 = (51, 52, 53, 54, 55) So n(n + 1) is divisible by 6.
For the required conditions having no two numbers, So n will be in the form of 6k, 6k + 2, 6k + 3 and
no two unit digits and no two ten’s digit same, we 6k + 5.
can select any 1 number from each of group 1, 2, 3, Case 1: If n in the form of 6k
4 and 5. First we can group the numbers in bundle of 6-6 num-
So total number of ways = 5C1 × 4C1 × 3C1 × 2C1 × 1C1 bers of the form 1, 2, 3, 4, 5, 6
= 5 × 4 × 3 × 2 × 1 = 120 So from each group we can select in subsets like [(1, 6),
Now if we divide 120 by 73 we get ‘47’ as a remainder. (2, 5), (3, 6)].
27. Given that a < b < c < d Case 2: If n is in the form of 6k + 2
b–a<c–b<d–c Then remaining 8 numbers 1, 2, 3, 4, 5, 6, 7, 8 can
So possibilities of a, b, c and d are be select in subsets like this [(1, 2, 3, 6), (4, 8), (5, 7)]
a = 1, b = 2, c = 4, d = 7 Case 3: If n is in the form of 6k + 3
a = 1, b = 2, c = 4, d = 8 Then remaining numbers 1, 2, 3, 4, 5, 6, 7, 8, 9 can be
a = 1, b = 2, c = 4, d = 9 select in subsets like this [(1, 2, 3, 6), (4, 5, 6), (7, 8)]
a = 1, b = 2, c = 5, d = 9 Case 4: When n is in the form of 6k + 5
a = 1, b = 2, c = 6, d = not possible Then remaining numbers 1 to 11 can be select in sub-
a = 2, b = 3, c = 5, d = 8 sets like this [(1, 10, 11), (5, 8, 9), (2, 3, 4, 6, 7)]
a = 2, b = 3, c = 5, d = 9 So we have 16 numbers of all these types from 4 to 99.
a = 2, b = 3, c = 6, d = not possible So total numbers are 16 × 4 = 64.
a = 3, b = 4, c = 6, d = 9
a = 4, b = 5, c = 7, d = not possible
So there are total 7, 4-digit numbers possible.

M05 PMRO -Math Olympiad XXXX 00_Solution.indd 44 12-Oct-20 11:18:50 AM

You might also like